You are on page 1of 21

' Distance Learning % &

! Dashboard Dashboard / My courses / 6 Surgery with pediatric surgery and oncology


/ 1 History and organization of surgical aid in Ukraine. Ethics and deontology in surgery. New technologies in surgery.
Modern methods of diagnosis and treatment.
! My Training / Test 1

" Catalog

# Calendar
Started on Sunday, 18 September 2022, 6:17 PM
$ Reports State Finished
Completed on Saturday, 15 October 2022, 6:41 PM
Time taken 27 days
" 6 Surgery with pediatric
surgery and oncology Marks 29.00/40.00
Grade 2.18 out of 3.00 (73%)
# Participants
Which English surgeon was outstanding for his efforts to introduce sys-tematic, scientifically based antisepsis in the
$ Badges treatment of wounds and the performance of surgical operations?

% Competencies Select one:


a. Frederick Treves (1853-1923)
& Grades
b. Robert Tait (1845-1899)
c. William Macewen (1848-1924)

d. Joseph Lister (1827-1912)

Which famous surgeon, notable for his method of controlling hemorrhage, humbly attributed his success with
patients to God, as noted in his famous motto, “Je le pansay. Dieu le guérit,” that is, “I treated him. God cured him.”

Select one:
a. Ambroise Paré (1510-1590)
b. William T. G. Morton (1819-1868)
c. Theodor Kocher (1841-1917)
d. John Hunter (1728-1793)

What statement about laparoscopic cholecystectomy is not correct?

Select one:
a. Since the gall bladder can’t be removed from the abdomen by suctioning out the bile through the 1 cm
incision at the patient's navel.
b. The length of postoperative stay in the hospital is minimal, and same-day discharges are possible in cases of
early morning procedures.
c. Laparoscopic cholecystectomy is the most common laparoscopic procedure performed.
d. Rather than a minimum 20 cm incision as in traditional (open) cholecystectomy, four incisions of 0.5–1.0 cm
will be sufficient to perform a laparoscopic removal of a gallbladder.
e. In this procedure, 5-10mm diameter instruments (graspers, scissors, and clip applier) can be introduced by
the surgeon into the abdomen through trocars (hollow tubes with a seal to keep the CO2 from leaking.

In response to the terrorist attacks on New York and Washington in Sep-tember 2001, evidence of public-
spiritedness is to be found in the Declara-tion of Professional Responsibility, subtitled Medicine's Social Contract
with Humanity. Which of the following declaration of the oath is NOT cor-rect?

Select one:
a. Apply our knowledge and skill when needed, although doing so may put us at risk.
b. Work freely with colleagues to discover, develop, and promote ad-vances in medicine and public health that
ameliorate suffering and contribute to human well-being.
c. Teach and mentor those who follow us for they are the future of our caring profession.
d. Supporting access to medical care for only the privileged

Replantation:

Select one:
a. Is a form of minimally invasive surgery that was designed to access many regions of the body via major blood
vessels
b. Is the reattachment of a completely detached body part.
c. Is joining damaged human tissues and restoration of the functioning of human organs. It is used to dissect
tissues and to stop or prevent bleeding when tissue and blood vessels are incised.
d. Is a modern surgical technique in which operations in the abdomen are performed through small incisions
(usually 0.5–1.5 cm) as compared to the larger incisions needed in laparotomy.
e. Is a general term for surgery requiring an operating microscope.

Which of the following local anesthetic agents has the longest duration of action?

Select one:
a. Tetracaine
b. Lidocaine
c. Prilocaine
d. Mepivacaine
e. Bupivacaine

What statement about Magnetic resonance imaging (MRI) is correct?

Select one:
a. It uses ionizing radiation in the radio frequency range.
b. Magnetic resonance imaging provides good spatial resolution (the ability to distinguish two separate
structures an arbitrarily small distance from each other)
c. Is a medical procedure in endoscopy is combined with ultrasound to obtain images of the internal organs in
the chest and abdomen.
d. Is imaging by sections or sectioning, through the use of any kind of penetrating wave.
e. Is a medical imaging technique used in radiology to visualize detailed internal structures that uses of nuclear
resonance for imaging

Consent is permission, granted by the patient to the surgeon, to make a diagnostic or therapeutic intervention on
the patient's behalf. All the fol-lowing are necessary for a consent to be valid EXCEPT:

Select one:
a. It must compel by force
b. It must be informed
c. It must be ethical

d. It must also be voluntary

What statement about Microsurgery is not correct?

Select one:
a. Microsurgery is a general term for surgery requiring an operating microscope.
b. Microsurgery allows transfer of tissue from one part of the body to another and re-attachment of severed
parts.
c. Microsurgery is not used in general surgery.
d. Microsurgery is used in ophthalmology, orthopedic surgery, gynecological surgery, otolaryngology,
neurosurgery, oral and maxillofacial surgery, and pediatric surgery.
e. Microsurgery allows regain smaller blood vessels and nerves (typically 1 mm in diameter)

1998 the American College of Surgeons adopted a Statement on Princi-ples Guiding Care at the End of Life. Which
of the following principles is INCORRECT?

Select one:
a. Recognize, assess, and address psychological, social, and spiritual problems
b. Use the most appropriate measures that are consistent with the choices of the patient or the patient's legal
surrogate.
c. Recognize the physician's responsibility to forgo treatments that are futile

d. Mystify the public and polity about present and future threats to the health of humanity.

Significant risks from laparoscopic surgery are all EXCEPT:

Select one:
a. Electrical burns of surrounding tissues.
b. Eventration
c. Pulmonary disorders in patients with pulmonary diseases
d. Trocar injuries of blood vessels

e. Trocar injuries of small or large bowel.

What does increase the risk of trocar injuries of small or large bowel during laparoscopic surgery?

Select one:
a. Malnutrition
b. Obesity
c. Adhesions (scar tissue) from previous abdominal surgery.
d. Two dimensional image of the laparoscope.

e. Coagulation disorders

What statement about Replantation is not correct?

Select one:
a. Fingers and thumbs are the most common but the ear, scalp, nose, face, arm and penis have all been
replanted.
b. Survival of the amputated piece was not enough to ensure success of the replant.
c. Generally replantation involves restoring blood flow through arteries and veins, restoring the bony skeleton
and connecting tendons and nerves as required.
d. Replantation is the reattachment of a completely detached body part.
e. If an amputated specimen is not able to be replanted to its original location entirely, this is means that the
specimen is unreplantable.

What statement about free tissue transfer is not correct?

Select one:
a. The vessels that supply the free flap are anastomosed with microsurgery to matching vessels (artery and
vein) in the reconstructive site.
b. The composite tissue is transferred (moved as a free flap of tissue) to the region on the patient requiring
reconstruction (e.g., mandible after oral cancer resection, breast after cancer resection, traumatic tissue loss,
congenital tissue absence).
c. Afer free tissue transfer the patients need immunosuppression.
d. Free tissue transfer is a surgical reconstructive procedure using microsurgery.
e. A region of "donor" tissue is selected that can be isolated on a feeding artery and vein; this tissue is usually a
composite of several tissue types (e.g., skin, muscle, fat, bone).

Which of the following is the MOST common complication in the surgical repair of blepharoptosis?

Select one:
a. Corneal abrasion

b. Undercorrection
c. Overcorrection
d. Exposure keratitis
e. Improperly positioned eyelid crease

Which of the following diseases is appropriately treated with combined heart-lung transplantation?

Select one:
a. Primary pulmonary hypertension

b. End-stage emphysema
c. End-stage pulmonary fibrosis secondary to sarcoidosis
d. Cystic fibrosis
e. Idiopathic dilated cardiomyopathy with long-standing secondary pulmonary hypertension

What statement about GammaKnife is not correct?:

Select one:
a. The GammaKnife system uses any kind of penetrating wave.

b. The Gamma Knife system does however require a head frame to be bolted onto the skull of the patient, and is
only capable of treating cranial lesions.
c. The GammaKnife system uses 201 Cobalt-60 sources located in a ring around a central treatment point
("isocenter").
d. The Gamma Knife system is equipped with a series of 4 collimators of 4mm, 8mm, 12mm and 16mm diameter,
and is capable of submillimeter accuracies.
e. Gamma Knife system is likely to be more accurate than Cyber Knife.

What statement about Welding technology is not correct?

Select one:
a. Welding technology is used to restoration of the functioning of human organs.
b. Welding technology is used with success for joining damaged human tissues.
c. Welding technology is used to dissect tissues.
d. Welding technology is used only for joining tissues.
e. Welding technology is used to stop or prevent bleeding when tissue and blood vessels are incised.

What statement about Robot-assisted surgery (Da Vinci) is not correct?:

Select one:
a. Da Vinci system allows greater precision and better visualization compared to standard laparoscopic surgery.
b. The physician operates two masters (similar to joysticks) that control the two mechanical arms on the robot.
The mechanical arms are armed with specialized instruments with hand-like movements which carry out the
surgery through tiny holes in the patient’s abdomen.
c. Da Vinci is a computer-assisted robotic system that expands a surgeon's capability to operate within the
abdomen in a less invasive way during laparoscopic surgery.
d. The surgeon is remote from the patient, working a few feet from the operating table while seated at a
computer console with a three-dimensional view of the operating field.
e. The operations with the Da Vinci System are performed with direct mechanical connection between the
surgeon and the patient.

Microsurgery:

Select one:
a. Is a form of minimally invasive surgery that was designed to access many regions of the body via major blood
vessels
b. Is joining damaged human tissues and restoration of the functioning of human organs. It is used to dissect
tissues and to stop or prevent bleeding when tissue and blood vessels are incised.
c. Is the reattachment of a completely detached body part.
d. Is a general term for surgery requiring an operating microscope.
e. Is a modern surgical technique in which operations in the abdomen are performed through small incisions
(usually 0.5–1.5 cm) as compared to the larger incisions needed in laparotomy.

Endovascular surgery is:

Select one:
a. Is joining damaged human tissues and restoration of the functioning of human organs. It is used to dissect
tissues and to stop or prevent bleeding when tissue and blood vessels are incised.
b. Is a modern surgical technique in which operations in the abdomen are performed through small incisions
(usually 0.5–1.5 cm) as compared to the larger incisions needed in laparotomy.
c. Is a form of minimally invasive surgery that was designed to access many regions of the body via major blood
vessels
d. Is the reattachment of a completely detached body part.
e. Is a general term for surgery requiring an operating microscope.

Which of the following surgeons reported the first successful ligation of a patent arterial duct in 1939 and resection
for coarctation of the aorta with direct anastomosis of the remaining ends in 1945?

Select one:
a. John Alexander (1891-1954)
b. Elliott Cutler (1888-1947)
c. Robert Gross (1905-1988)
d. laude Beck (1894-1971)

What is contra-indication for laparoscopic cholecystectomy?

Select one:
a. Acute cholecystitis

b. Cholangitis
c. Choledocholithiasis
d. Large stone in gallbladder
e. Adhesions in upper part of abdome

What statement about Cyber Knife is not correct?:

Select one:
a. The Cyber Knife is a frameless robotic radiosurgery system.
b. The element of the CyberKnife is a robotic arm which allows the energy to be directed at any part of the body
from any direction.
c. The element of the CyberKnife is the radiation produced from a small linear particle accelerator

d. Treatment with CyberKnife system does not require real time imaging capability as the frame does not allow
movement during treatment.
e. The CyberKnife system is a method of delivering radiotherapy, with the intention of targeting treatment more
accurately than standard radiotherapy.

In centers with experienced personnel, 1-year liver transplant survival is now approximately

Select one:
a. 50%
b. 80%
c. 65%
d. 35%
e. 95%

The hollowed-out appearance following cosmetic blepharoplasty is MOST commonly associated with

Select one:
a. Poor wound healing and scarring
b. Excesive fat resection
c. Excesive of muscle resection
d. Excesive of skin resection
e. Damage to the lacrimal gland

After the first postoperative year of cardiac transplantation, the most common cause of death is:

Select one:
a. Acute rejection episode
b. Arrhythmia
c. Cancer
d. Accelerated graft arteriosclerosis
e. Infection

At the beginning of the 20th century, Alexis Carrel developed a new revolutionary technique. What was it?

Select one:
a. New technique to suture a wound that penetrated a cardiac chamber

b. New suturing technique to anastomose the smallest of blood vessels


c. New technique of creating immunosuppression as a method to allow the host to accept the foreign transplant
d. New technique for mammaplasty

Welding is:

Select one:
a. Is a modern surgical technique in which operations in the abdomen are performed through small incisions as
compared to the larger incisions needed in laparotomy.
b. Is a general term for surgery requiring an operating microscope.
c. Is the reattachment of a completely detached body part.

d. Is a form of minimally invasive surgery that was designed to access many regions of the body via major blood
vessels
e. Is joining damaged human tissues and restoration of the functioning of human organs. It is used to dissect
tissues and to stop or prevent bleeding when tissue and blood vessels are incised.

The MOST appropriate management of spider telangiectasia is

Select one:
a. Chemical peeling
b. Excision
c. Incisional biopsy
d. Electrocautery
e. CO2 laser

What statement about CT is not correct?

Select one:
a. CT scanning is not contraindicated in case of insulin pumps.
b. CT provides comparable resolution with far better contrast resolution (the ability to distinguish the
differences between two arbitrarily similar but not identical tissues).
c. It uses ionizing radiation
d. CT provides good spatial resolution (the ability to distinguish two separate structures an arbitrarily small
distance from each other)
e. CT scanning is not contraindicated in case of cardioverter-defibrillator.

Laparoscopic surgery is:

Select one:
a. Is the reattachment of a completely detached body part.
b. Is joining damaged human tissues and restoration of the functioning of human organs. It is used to dissect
tissues and to stop or prevent bleeding when tissue and blood vessels are incised.
c. Is a form of minimally invasive surgery that was designed to access many regions of the body via major blood
vessels
d. Is a modern surgical technique in which operations in the abdomen are performed through small incisions as
compared to the larger incisions needed in laparotomy.
e. Is a general term for surgery requiring an operating microscope.

The major limitations of standard laparoscopic techniques are following EXCEPT:

Select one:
a. Two dimensional image of the laparoscope.

b. Limitation of instruments movement.


c. Adhesions (scar tissue) from previous abdominal surgery.
d. Retraction of internal organs.
e. Bipolar coagulator.

The first successful open heart operation in 1953, conducted with the use of a heart-lung machine, was a
momentous surgical contribution. Which of these surgeons devised the machine?

Select one:
a. Charles Bailey (1910-1993)

b. Russell Brock (1903-1980)


c. Luther Hill (1862-1946)
d. John Gibbon (1903-1973)

In an exciting era for surgeons, what important surgical procedure did Al-len Oldfather Whipple (1881-1963)
introduce in 1935?

Select one:
a. Pancreaticoduodenectomy for cancer of the pancreas
b. Vagotomy for operative treatment of peptic ulcer

c. Splanchnic resection for hypertension


d. Intracranial section of various cranial nerves

Graft-versus-host disease has occurred with the transplantation of which of the following?

Select one:
a. Lung
b. Heart
c. Bone marrow
d. Pancreas

e. Kidney

There are advantages to the patient with laparoscopic surgery versus an open procedure EXCEPT:

Select one:
a. Hospital stay is less, and often with a same day discharge which leads to a faster return to everyday living.
b. Smaller incision, which reduces pain and shortens recovery time, as well as resulting in less post-operative
scarring.
c. Laparoscopic surgery is more suitable for patients with pulmonary diseases.
d. Less pain, leading to less pain medication needed.
e. Reduced exposure of internal organs to possible external contaminants thereby reduced risk of acquiring
infections.

You are excising a scar by the fusiform ellipse thecnique. The excision line is 1 cm wide. What is the value of "X"
representing the length of the fusiform incision?

Select one:
a. 5 cm

b. 2 cm
c. 3 cm
d. 4 cm
e. 1 cm

In 1900, which surgeon described his technique for a suprapubic surgical incision?

Select one:
a. John Finney (1863-1942)

b. Hermann Pfannenstiel (1862-1909)


c. Hugh Hampton Young (1870-1945)
d. Friedrich Trendelenburg (1844-1924)

Which of the following statements regarding heart transplantation is true?

Select one:
a. Cold ischemia time for donor hearts should not be more than 48 h
b. Cadaveric graft survival is significantly lower with heart transplants as compared with renal transplants
c. The upper age limit for heart transplant eligibility is 55 years

d. The leading cause of death after the first year of cardiac transplantation is chronic rejection
e. Heart transplants are matched by size and ABO blood type rather than tissue typing

Finish review

◀︎ Forum Jump to... 01.1 ▶︎

Quiz navigation
1 2 3 4 5 6 7 8 9 10 11 12 13 14 15 16 17 18 19 20 21 22 23

24 25 26 27 28 29 30 31 32 33 34 35 36 37 38 39 40

Show one page at a time


Finish review

Copyright © 2022 Technomatix Ltd. | All Rights Reserved


' Distance Learning % &

! Dashboard Dashboard / My courses / 6 Surgery with pediatric surgery and oncology


/ 2 Systemic inflammatory response syndrome (SIRS) in surgical patients. Pathogenesis and significance in different
diseases and traumas. Rules of antibiotic therapy in the prevention and treatment of surgical infection.
! My Training / Test 2

" Catalog

# Calendar
Started on Monday, 19 September 2022, 6:24 PM
$ Reports State Finished
Completed on Monday, 19 September 2022, 7:50 PM
Time taken 1 hour 26 mins
" 6 Surgery with pediatric
surgery and oncology Marks 14.00/20.00
Grade 2.10 out of 3.00 (70%)
# Participants
A 24 y.o. woman consulted a doctor about continued fever, night sweating. She lost 7 kg within the last 3 months.
$ Badges She had casual sexual contacts. Objectively: enlargement of all lymph nodes, hepatolienal syndrome. Blood count:
leukocytes - 2,2*109/L. What disease can be suspected?
% Competencies
Select one:
& Grades a. Lymphogranulomatosis
b. Tuberculosis
c. HIV-infection

d. Infectious mononucleosis
e. Chroniosepsis

A 40-year-old patient underwent an operation for a lumbar phlegmon. Body temperature rose again up to 38oC, he
got intoxication symptoms, there was an increase of leukocyte number in blood. The wound that was nearly free
from necrotic tissues and full of granulations started to discharge pus, the granulations turned pale. What
complication developed in this patient?

Select one:
a. Erysipeloid
b. Allergic reaction
c. Erysipelas

d. Putrid phlegmon
e. Sepsis

Major alterations in pulmonary function associated with adult respiratory distress syndrome (ARDS) include

Select one:
a. Hypoxemia
b. Increased resting lung volume
c. Increased pulmonary compliance
d. Decreased dead space ventilation

e. Increased functional residual capacity

A 52 year old patient complains about pain in the right part of her chest, dyspnea, and cough with a lot of foul-
smelling albuminoid sputum in form of "meat slops". Objectively: the patient's condition is grave, cyanosis is
present, breathing rate is 31/min, percussion sound above the right lung is shortened, auscultation revealed different
rales. What is the most probable diagnosis?

Select one:
a. Lung gangrene
b. Chronic pneumonia
c. Multiple bronchiectasis
d. Lung abscess

e. Pleura empyema

A 42-year-old man has a calculated resting energy expenditure of 1800 kcal/day (basal energy expenditure plus
10%). Match the following clinical situations with the appropriate daily energy requirement. In case of sepsis energy
expenditure is

Select one:
a. 1600
b. 2800
c. 4500
d. 2300

e. 3600

Which of the following surgical interventions is least likely to provide acceptable prolongation of life for patients with
AIDS?

Select one:
a. Colonic resection for perforation secondary secondary to cytomegalovirus infection
b. Splenectomy for AIDS-related idiopathic thrombocytopenic purpura
c. Cholecystectomy for acalculous cholecystitis
d. Gastric resection for a bleeding gastric lymphoma or Kaposi’s sarcoma
e. Tracheostomy for ventilator dependent patients with respiratory failure

Central venous pressure (CVP) may be decreased by

Select one:
a. Pneumothorax
b. Positive-pressure ventilation
c. Gram-negative sepsis
d. Hypervolemia
e. Pulmonary embolism

Which statement about transmission of HIV in the health care setting is true?

Select one:
a. All needles should be capped immediately after use
b. Double gloving reduces the risk of intraoperative needle sticks
c. The risk of seroconversion following a needle stick with a contaminated needle is greater for HIV than for
hepatitis B
d. A freshly prepared solution of dilute chlorine bleach will not adequately decontaminate clothing
e. Cuts and other open skin wounds are believed to act as portals of entry for HIV

On the 4th day after recovering from a cold a patient was hospitalized with complaints of solitary spittings of mucoid
sputum. On the 2nd day there was a single discharge of about 250 ml of purulent blood-streaked sputum.
Objectively: the patient's condition is moderately severe. Respiratory rate - 28-30/min, Ps- 96 bpm, AP- 110/70 mm
Hg. Respiration above the left lung is vesicular, weak above the right lung. There are moist rales of different types
above the lower lobe and amphoric breath near the angle of scapula. What is the most likely diagnosis?

Select one:
a. Exudative pleuritis

b. Acute pulmonary abscess


c. Pleural empyema
d. Pyopneumothorax
e. Acute focal pneumonia

Which statement regarding transmission of viral illness through homologous blood transfusion is true?

Select one:
a. The most frequent infectious complication of blood transfusion continues to be viral meningitis
b. The most common viral agent transmitted via blood transfusion in the United States is human immune
deficiency virus (HIV)
c. Blood is routinely tested for cytomegalovirus (CMV) because CMV infection is often fatal

d. The etiologic agent in post transfusion hepatitis remains undiscovered


e. Up to 10% of those who develop post transfusion hepatitis will develop cirrhosis or hepatoma or both

A 30-year-old patient complains of pain, hyperemia along subcutaneous veins, rise in body temperature. While
examining the large shin subcutaneous vein, there is hyperemia, pain by pressing. Homanss and Luses's symptoms
are negative. What is the preliminary diagnosis?

Select one:
a. Acute ileofemoral phlebothrombosis
b. Thrombosis of aorta
c. Lymphostasis

d. Embolism of aorta
e. Acute thrombophlebitis of subcutaneous veins

A 24-year-old law student is brought to the emergency room complaining of severe abdominal pain of 6-8 hours
duration. He had been to a party the night before. The pain is in the epigastrium radiating to the back and is
accompanied by nausea. The patient had vomited twice prior to coming to the emergency room. Clinical examination
revealed that the young man was anxious, with acute condition, with a regular pulse rate of 100/min, blood pressure
of 100/68 mm Hg, and body temperature of 38,1oC. The most likely diagnosis is:

Select one:
a. Acute pancreatitis
b. Acute cholecystitis
c. Acute appendicitis

d. Acute diverticulitis
e. Mesenteric adenitis

A 65 y.o. man who has problems with urination as a result of benign prostate gland adenoma developed fever and
chill, hypotension, sinus tachycardia. Skin is warm and dry. Clinical blood analysis revealed absolute
granulocytopenia. These hemodynamic changes are most likely to be caused by:

Select one:
a. Secondary circulation insufficiency with retained systolic function as a result of peripheral vasoconstriction
b. Secondary reflex vasodilatation as a result of lowered cardiac output
c. Reflex vagus stimulation with lowered cardiac output
d. Endotoxemia with activation of complement system

e. Secondary endothelial changes as a result of bacterial lesion

A patient complains of an extremely intense pain in epigastrium. He has peptic ulcer disease of duodenum for 10
years. The patient is in the forced position being on the right side with legs abducted to stomach. Abdomen has
acute tenderness in the epigastrium. Guarding contraction of the abdominal wall muscles is observed. What is the
preliminary diagnosis?

Select one:
a. Acute condition of peptic ulcer disease
b. Penetration of ulcer into pancreas
c. Thrombosis of mesenteric vessels
d. Acute pancreatitis

e. Perforation of ulcer

A 20 y.o. patient suddenly felt ill 12 hours ago. There was pain in epigastric area, nausea, sporadic vomiting. He had
taken alcohol before. In few hours the pain localized in the right iliac area. On examination: positive rebound
tenderness symptoms. WBC- 12,2·109/L. What is the most probable diagnosis?

Select one:
a. Acute cholecystitis
b. Perforated ulcer
c. Acute appendicitis
d. Acute pancreatitis

e. Right side kidney colic

Human immunodeficiency virus (HIV) has been isolated from many body fluids. Which of the following is a major
source of transmission?

Select one:
a. Breast milk
b. Urine
c. Tears
d. Semen
e. Sweat

Signs and symptoms associated with early sepsis include

Select one:
a. Respiratory acidosis
b. Decreased cardiac output
c. Hypoglycemia
d. Increased arteriovenous oxygen difference
e. Cutaneous vasodilation

The appropriate antibiotic to prescribe while awaiting specific culture verification is

Select one:
a. Penicillin
b. Azathioprine
c. Erythromycin
d. Tetracycline
e. Cloxacillin

A 33 y.o. patient was admitted to the reception room of the Central District Hospital. He complains of a severely
painful swelling localized on posterior neck, fever up to 38,40C and general weakness. In anamnesis: diabetes
mellitus within 5 years. On physical examination on the posterior neck surface there is an infiltrate elevated above
surrounding skin. The tissues affected by swelling are tense and blue reddish discoloration in central area. There are
also several purulent necrotic pustules which are connected with each other and form a large skin necrosis. A
thinned necrotic skin of this swelling has holes looking like sieve, pus discharges throughout. What disease should a
doctor consider first of all?

Select one:
a. Acute skin cellulitis
b. Carbuncle associated with anthrax
c. Carbuncle
d. Furuncle
e. Skin abscess

Tissue injury or infection results in the release of tumor necrosis factor (TNF) by which of the following cells?

Select one:
a. Damaged vascular endothelial cells

b. Activated killer lymphocytes


c. Monocytes/macrophages
d. Activated T lymphocytes
e. Fibroblasts

Finish review

◀︎ 01.2 Jump to... 02 ▶︎

Quiz navigation
1 2 3 4 5 6 7 8 9 10 11 12 13 14 15 16 17 18 19 20

Show one page at a time


Finish review

Copyright © 2022 Technomatix Ltd. | All Rights Reserved


' Distance Learning % &

! Dashboard Dashboard / My courses / 6 Surgery with pediatric surgery and oncology


/ 3 Critical conditions in surgical clinic: circulatory collapse, acute respiratory distress syndrome, abdominal compartment
syndrome. Shock in surgical patients. Causes, diagnosis, therapeutic tactics.
! My Training / Test 3

" Catalog

# Calendar
Started on Saturday, 15 October 2022, 10:27 AM
$ Reports State Finished
Completed on Saturday, 15 October 2022, 10:41 AM
Time taken 13 mins 41 secs
" 6 Surgery with pediatric
surgery and oncology Marks 20.00/20.00
Grade 3.00 out of 3.00 (100%)
# Participants
A full-term male newborn experiences respiratory distress immediately after birth. A prenatal sonogram had been
$ Badges read as normal. An emergency radiograph is shown below. The patient was intubated and placed on 100% O2. The
arterial blood gas revealed pH 7.24; PO2 60 kPa; PCO2 52 kPa. The baby has sternal retractions and a scaphoid
% Competencies abdomen. Which of the following statements correctly refers to this condition?

& Grades Select one:


a. The most important aspect in management would be immediate exploration and repair of the defect.
b. The defect is usually anteromedial in location.
c. Any abdominal organ can be involved.

d. The size of the defect directly correlates with severity of the disease.
e. The most likely cause of this problem is in utero traumatic rupture of the diaphragm.

When operating to repair civilian colon injuries (choose the correct statement)

Select one:
a. The presence of shock on admission or more than two associated intraabdominal injuries is an absolute
contraindication to primary colonic repair.
b. Administration of intravenous antibiotics with aerobic and anaerobic coverage has not been shown to
decrease the incidence of wound infections after repair of colonic injuries.
c. Right-sided colonic wounds should not be repaired primarily.
d. A colostomy should be performed for colonic injury in the presence of gross fecal contamination.
e. Distal sigmoidal injuries should not be repaired primarily.

Which of the following parts of the body deteriorates the fastest without constant perfusion?

Select one:
a. Skeletal muscle.
b. Kidneys.
c. Skin
d. Heart.
e. Liver.

Which of the following terms best describes the failure of the circulatory system to provide sufficient circulation so
that each body part can perform it functions?

Select one:
a. Shock.

b. Epistaxis.
c. Oxygenation.
d. Distress syndrome.
e. Perfusion.

Animal and clinical studies have shown that administration of lactated Ringer’s solution to patients with hypovolemic
shock may

Select one:
a. Impair liver function.

b. Increase serum lactate concentration.


c. Increase metabolic acidosis.
d. Improve hemodynamics by alleviating the deficit in the interstitial fluid compartment.
e. Increase the need for blood transfusion.

Which of these hernias can most likely cause acute respiratory distress in infants?

Select one:
a. Foramen of Morgagni hernia.

b. Sliding hiatal hernia.


c. Rupture of the diaphragm.
d. Paraesophageal hiatal hernia.
e. Foramen of Bochdalek hernia.

The principle ways of assessing intraabdominal pressure are the following EXCEPT:

Select one:
a. peritoneal catheter.
b. urinary bladder catheter .

c. vaginal catheter.
d. gastric balloon.
e. superior vena cava catheter.

A conscious person in shock should be given:

Select one:
a. coffee or tea.
b. nothing by mouth.

c. clear liquids.
d. alcohol.
e. orange juice.

The clinical signs of ACS don’t include:

Select one:
a. decreased PO2.
b. lack of urinary output.

c. high peak expiratory pressure.


d. very highly elevated PCO2.
e. a massively distended abdomen.

Cardiogenic shock develops when:

Select one:
a. too much blood is lost..
b. sympathetic vascular tone is lost..
c. the heart no longer pumps effectively.

d. the volume of vascular fluid is reduced.


e. the lungs no longer take in oxygen.

Treatment of a patient in anaphylactic shock should include:

Select one:
a. intramuscular injection of nitroglycerin.
b. intramuscular injection of sympathomimetics.
c. immersion in an ice bath.

d. vigorous massage of the extremities.


e. administration of oxygen.

Signs and symptoms associated with early sepsis include

Select one:
a. Cutaneous vasodilation.
b. Increased arteriovenous oxygen difference.
c. Hypoglycemia.

d. Respiratory acidosis.
e. Decreased cardiac output.

Which of the following signs and symptoms suggest possible hypovolemic shock?

Select one:
a. Weak, rapid pulse; cold, clammy skin; pallor; shallow breathing.
b. Slow, strong pulse; dizziness; cold perspiration; nausea.
c. Blank expression; cold extremities; regular breathing.
d. Blank expression; chills; unconsciousness; dry skin.

e. Rapi pulse and normal arterial oressure.

Psychogenic shock can be caused by:

Select one:
a. intramuscular injection of sympathomimetics
b. loss of blood or body fluid.
c. an allergic reaction to insect bites.
d. temporary, generalized vascular dilation.

e. damage to the cervical spine.

The response to shock includes which of the following metabolic effects?

Select one:
a. Hyperkalemia.
b. Hypoglycemia.
c. Increase in sodium and water excretion.
d. Decrease in cortisol levels.

e. Increase in renal perfusion.

The use of oxygen in treatment of shock is considered to be:

Select one:
a. useful only if the patient has difficulty breathing.
b. of no particular benefit to the patient.
c. a part of routine treatment.
d. of a benefit in patients with AIRDS.
e. a last resort measure if all other treatment fails.

Choose the correct statement concerning the use of the pneumatic antishock garment (PASG)

Select one:
a. Should be terminated by means of prompt deflation as soon as the trauma patient reaches the emergency
department.
b. Is not recommended for control of persistent bleeding in the setting of severe pelvic fracture.
c. Expedites assessment of lower body injuries in the trauma patient.
d. Elevates blood pressure by an “autotransfusion” effect, with augmentation of venous return and cardiac
output.
e. Increases peripheral vascular resistance.

Which of the following statements regarding stress ulceration is true?

Select one:
a. It classically involves the antrum.
b. It is true ulceration, extending into and through the muscularis mucosa.

c. Increased secretion of gastric acid has been shown to play a causative role.
d. It frequently involves multiple sites.
e. It is seen following shock or sepsis, but for some unknown reason does not occur following major surgery,
trauma, or burns.

Major alterations in pulmonary function associated with adult respiratory distress syndrome (ARDS) include

Select one:
a. Increased functional residual capacity.
b. Increased pulmonary compliance.
c. Decreased dead space ventilation.

d. Increased resting lung volume.


e. Hypoxemia.

Predisposing factors for the development of intraabdominal hypertension include the following EXCEPT:

Select one:
a. acidosis.
b. polytransfusion.
c. dilutional coagulopathy or disseminated intravascular coagulation .
d. hypothermia.

e. spinal cord injury.

Finish review

◀︎ 02 Jump to... 03.1 ▶︎

Quiz navigation
1 2 3 4 5 6 7 8 9 10 11 12 13 14 15 16 17 18 19 20

Show one page at a time


Finish review

Copyright © 2022 Technomatix Ltd. | All Rights Reserved


( Distance Learning % &

" Dashboard Dashboard / My courses / 6 Surgery with pediatric surgery and oncology
/ 4 Abdominal pain, estimation of pain syndrome features at differential diagnostics of abdominal viscus diseases / Test 4

! My Training

" Catalog
Started on Sunday, 2 October 2022, 9:09 AM
# Calendar State Finished
Completed on Sunday, 2 October 2022, 9:14 AM
$ Reports
Time taken 5 mins 2 secs
Marks 20.00/20.00
# 6 Surgery with pediatric Grade 3.00 out of 3.00 (100%)
surgery and oncology
A 22-year-old man develops anorexia followed by vague periumbilical pain
$ Participants that several hours later becomes sharp, severe, constant, and well localized to
the right lower quadrant of the abdomen. He has abdominal tenderness, guarding, and rebound to the right and
% Badges below the umbilicus, temperature of 37,8°C, and white blood cell (WBC) count is 12,500, with neutrophilia and
immature forms. What is it?
& Competencies
Select one:
' Grades a. Acute pancreatitis
b. Acute cholecystitis
c. Acute diverticulitis

d. Acute appendicitis !

e. Strangulated obstruction

The correct answer is: Acute appendicitis

A 23-year-old woman describes exquisite pain with defecation and blood


streaks on the outside of the stools. Because of the pain she avoids having
bowel movements and when she finally does, the stools are hard and even
more painful. Physical examination cannot be done, as she refuses to allow
anyone to even "spread her cheeks" to look at the anus for fear of precipitating
the pain.

Select one:
a. Strangulated obstruction
b. Crohn’s disease
c. Anal fissure !

d. Ureteral colic
e. Ischiorectal abscess

The correct answer is: Anal fissure

A 24-year-old woman develops moderate, generalized abdominal pain of sudden onset, and shortly thereafter
faints. At the time of evaluation in the ER she is pale, tachycardic, and hypotensive. The abdomen is mildly distended
and tender, and she has hemoglobin of 70 g/l. There is no history of trauma. On inquiring as to whether she might be
pregnant, she denies the possibility because she has been on birth control pills since she was 14, and has never
missed taking them. What is it?

Select one:
a. Torsion of an ovarian cyst
b. Acute cholecystitis
c. Bleeding from a ruptured hepatic adenoma !

d. Acute pancreatitis

e. Obstructive jaundice

The correct answer is: Bleeding from a ruptured hepatic adenoma

A 27-year-old man is recovering from an appendectomy for gangrenous acute appendicitis with perforation and
periappendicular abscess. He has been receiving Clindamycin and Tobramycin for 7 days. Eight hours ago he
developed watery diarrhea, crampy abdominal pain, fever, and leukocytosis. What is it?

Select one:
a. Pseudomembranous colitis !

b. Cancer of sigmoid
c. Torsion of an ovarian cyst
d. Obstructive jaundice

e. Strangulated obstruction

The correct answer is: Pseudomembranous colitis

A 28-year-old man is brought to the office by his mother. Beginning 4 months ago he has had three operations,
done elsewhere, for a perianal fistula, but after each one the area has not healed, and in fact the surgical wounds
have become bigger. He now has multiple non healing ulcers, fissures, and fistulas all around the anus, with purulent
discharge. There are no palpable masses. What is it?

Select one:
a. Torsion of an ovarian cyst
b. Ureteral colic
c. Crohn’s disease !

d. Ischiorectal abscess

e. Strangulated obstruction

The correct answer is: Crohn’s disease

A 33-year-old alcoholic man shows up in the ER with epigastric and midabdominal pain that began 12 hours ago
shortly after the ingestion of a large meal. The pain is constant and very severe, and radiates straight through to the
back. He vomited twice early on, but since then has continued to have retching. He has tenderness and some
muscle guarding in the upper abdomen, is afebrile, and has mild tachycardia. Serum amylase is 6,5, and his
hematocrit is 52%. What is it?

Select one:
a. Cancer of sigmoid
b. Obstructive jaundice
c. Strangulated obstruction
d. Acute edematous pancreatitis !

e. Torsion of an ovarian cyst

The correct answer is: Acute edematous pancreatitis

A 44-year-old alcoholic man presents with severe epigastric pain that began shortly after a heavy bout of alcoholic
intake, and reached maximum intensity over a period of 2 hours. The pain is constant, radiates straight through to
the back, and is accompanied by nausea, vomiting, and retching. He had a similar episode 2 years ago, for which he
required hospitalization. The best tactics option in this patient would be:

Select one:
a. Peritoneal lavage
b. Diagnostic laparotomy
c. X-ray abdominal cavity
d. Endoscopic retrograde cholangiopancreatography
e. Sonogram, elective cholecystectomy will follow !

The correct answer is: Sonogram, elective cholecystectomy will follow

A 44-year-old man shows up in the ER at 11 PM with exquisite perianal pain. He cannot sit down, reports that bowel
movements are very painful, and has been having chills and fever. Physical examination shows a tender, red,
fluctuant mass between the anus and the ischial tuberosity.

Select one:
a. Acute appendicitis
b. Ischiorectal abscess !

c. Strangulated obstruction
d. Ureteral colic
e. Torsion of an ovarian cyst

The correct answer is: Ischiorectal abscess

A 48-year-old woman develops pain of the right lower quadrant while playing tennis. The pain progresses and the
patient presents to the emergency room later that day with a low-grade fever, a white blood count of 13,000, and
complaints of anorexia and nausea as well as persistent, sharp pain of the right lower quadrant. On examination she
is tender in the right lower quadrant with muscular spasm and there is a suggestion of a mass effect. An ultrasound
is ordered and shows an apparent mass in the abdominal wall. Which of the following is the most likely diagnosis?

Select one:
a. Hematoma of the rectus sheath !

b. Caecal carcinoma
c. Cholecystitis
d. Acute appendicitis
e. Torsion of an ovarian cyst

The correct answer is: Hematoma of the rectus sheath

A 52-year-old man has right flank colicky pain of sudden onset that radiates
to the inner thigh and scrotum. There is microscopic hematuria. What is it?

Select one:
a. Strangulated obstruction

b. Rupture of abdominal aorta


c. Torsion of an ovarian cyst
d. Ureteral colic !

e. Acute appendicitis

The correct answer is: Ureteral colic

A 52-year-old obese mother of three children presents with progressive jaundice, which she first noticed 6 weeks
ago. She has a total bilirubin of 107, with 98 direct and 9 indirect, and minimally elevated SGOT. The alkaline
phosphatase is about six times the upper limit of normal. She gives a history of multiple episodes of colicky right
upper quadrant abdominal pain, brought about by ingestion of fatty food. The initial maneuver should be to:

Select one:
a. Endoscopic retrograde cholangiopancreatography !

b. Peritoneal lavage
c. X-ray examination of the abdominal cavity
d. Diagnostic laparotomy
e. Sonogram, elective cholecystectomy will follow

The correct answer is: Endoscopic retrograde cholangiopancreatography

A 54-year-old man has had colicky abdominal pain and protracted vomiting for
several days. He has developed progressive moderate abdominal distention,
and has not had a bowel movement or passed any gas for 5 days. He has high-
pitched, loud bowel sounds that coincide with the colicky pain, and x-rays
show distended loops of small bowel and air-fluid levels. Five years ago he had
an exploratory laparotomy for a gunshot wound of the abdomen. What is it?

Select one:
a. Mechanical intestinal obstruction, caused by adhesion !

b. Acute appendicitis
c. Torsion of an ovarian cyst
d. Cecal carcinoma
e. Hematoma of the rectus sheath

The correct answer is: Mechanical intestinal obstruction, caused by adhesion

A 54-year-old man has had colicky abdominal pain and protracted vomiting for several days. He has developed
progressive moderate abdominal distention,
and has not had a bowel movement or passed any gas for 5 days. He has high-
pitched, loud bowel sounds that coincide with the colicky pain, and x-rays
show distended loops of small bowel and air-fluid levels. Five years ago he had
an exploratory laparotomy for a gunshot wound of the abdomen. Six hours
after being hospitalized and placed on NG suction and IV fluids, he develops
fever, leukocytosis, abdominal tenderness, and rebound tenderness. What is it?

Select one:
a. Acute pancreatitis
b. Acute appendicitis

c. Cecal carcinoma
d. Torsion of an ovarian cyst
e. Strangulated obstruction !

The correct answer is: Strangulated obstruction

A 54-year-old obese man gives a history of burning retrosternal pain and heartburn that is brought about by
bending over, wearing tight clothing, or lying flat in bed at night. He gets symptomatic relief from antacids but has
never been formally treated. The problem has been present for many years, and seems to be progressing. What is
it?

Select one:
a. Gastric peptic ulcer
b. Gastro esophageal reflux disease !

c. Acute mesenteric ischemia


d. Acute pancreatitis
e. Acute cholecystitis

The correct answer is: Gastro esophageal reflux disease

A 56-year-old man presents with progressive jaundice, which he first noticed 6 weeks ago. He has a total bilirubin of
225, with 165 direct and 60 indirect, and minimally elevated SGOT. The alkaline phosphatase is about eight times
the upper limit of normal. He has lost 5 kg over the past 2 months, and has a persistent, nagging mild pain deep into
his epigastrium and in the upper back. His sister died at age 44 from a cancer of the pancreas. A sonogram shows
dilated intrahepatic ducts, dilated extrahepatic ducts, and a much distended, thin-walled gallbladder.

Select one:
a. Obstructive jaundice
b. Cancer of the head of the pancreas !

c. Acute ascending cholangitis


d. Hepatocellular jaundice
e. Acute edematous pancreatitis

The correct answer is: Cancer of the head of the pancreas

A 59-year-old woman has a history of three prior episodes of left lower quadrant abdominal pain for which she was
briefly hospitalized and treated with antibiotics. She began to feel discomfort 12 hours ago, and now she has
constant left lower quadrant pain, tenderness, and a vaguely palpable mass. She has fever and leukocytosis. What is
it?

Select one:
a. Acute appendicitis
b. Cecal carcinoma
c. Strangulated obstruction

d. Acute diverticulitis !

e. Torsion of an ovarian cyst

The correct answer is: Acute diverticulitis

A 66-year-old man has an upper Gl endoscopy done as an outpatient to check on the progress of medical therapy
for gastric ulcer. Six hours after the procedure, he returns complaining of severe, constant retrosternal pain that
began shortly after he went home. He looks prostrate and very ill, is diaphoretic, has a fever of 38°C, and a
respiratory rate of 30. There is a hint of subcutaneous emphysema at the base of the neck. What is correct
diagnosis?

Select one:
a. Acute mesenteric thrombosis
b. Instrumental perforation of the esophagus !

c. Right-side lobar pneumonia

d. Perforated duodenum ulcer


e. Acute pancreatitis

The correct answer is: Instrumental perforation of the esophagus

A 73-year-old obese mother of six children has severe right upper quadrant abdominal pain that began 3 days ago.
The pain was colicky at first but has been constant for the past 2.5 days. She has tenderness to deep palpation,
muscle guarding, and rebound in the right upper quadrant. She has temperature spikes of 38,4 and 38,6°C, with
chills. Her WBC count is 22,000, with a shift to the left. Her bilirubin is 39µmol/l, and she has an alkaline
phosphatase of 2,000 (about 20 times more than normal).

Select one:
a. Acute ascending cholangitis !

b. Perforated duodenum ulcer


c. Hepatocellular jaundice

d. Torsion of an ovarian cyst


e. Acute pancreatitis

The correct answer is: Acute ascending cholangitis

A white, obese 40-year-old mother of five children gives a history of repeated episodes of right upper quadrant
abdominal pain brought about by the ingestion of fatty foods, and relieved by the administration of anticholinergic
medications. The pain is colicky, radiates to the right shoulder and around to the back, and is accompanied by
nausea and occasional vomiting. Physical examination is unremarkable. What is it?

Select one:
a. Gallstones, with biliary colic !

b. Acute appendicitis
c. Hematoma of the rectus sheath
d. Caecal carcinoma

e. Torsion of an ovarian cyst

The correct answer is: Gallstones, with biliary colic

An 82-year-old man develops severe abdominal distension, nausea, vomiting, and colicky abdominal pain. He has
not passed any gas or stool for the past 12 hours. He has a tympanic abdomen with hyperactive bowel sounds. X-ray
shows distended loops of small and large bowel, and a very large gas shadow that is located in the right upper
quadrant and tapers toward the left lower quadrant with the shape of a parrot's beak. What is it?

Select one:
a. Volvulus of the sigmoid !

b. Gallstones, with biliary colic


c. Acute appendicitis
d. Cecal carcinoma

e. Torsion of an ovarian cyst

The correct answer is: Volvulus of the sigmoid

Finish review

◀︎ 03.3 Jump to... 04 ▶︎

Quiz navigation
1 2 3 4 5 6 7 8 9 10 11 12 13 14 15 16 17 18 19 20

Show one page at a time


Finish review

Copyright © 2022 Technomatix Ltd. | All Rights Reserved


( Distance Learning % &

" Dashboard Dashboard / My courses / 6 Surgery with pediatric surgery and oncology
/ 5 Syndromes of dyspepsia, vomiting and violation of defecation. Their value in diagnosis and differential diagnosis. Colitis
syndrome.
! My Training / Test 5

" Catalog

# Calendar
Started on Sunday, 2 October 2022, 9:15 AM
$ Reports State Finished
Completed on Sunday, 2 October 2022, 9:17 AM
Time taken 2 mins 38 secs
# 6 Surgery with pediatric
surgery and oncology Marks 20.00/20.00
Grade 3.00 out of 3.00 (100%)
$ Participants
A 42-year-old woman is admitted to the emergency department with severe colicky pain, vomiting, and abdominal
% Badges distention. She has not passed stools
or flatus for 48 hours. X-rays of the abdomen confirm the presence of smallbowel obstruction. What is the most
& Competencies likely cause of small-bowel obstruction
in this patient?
' Grades
Select one:
a. Ulcerative colitis
b. Gallstone ileus
c. Adhesions !

d. Crohn’s disease
e. Adenocarcinoma

The correct answer is: Adhesions

All of statements about bowel obstruction are correct EXCEPT:

Select one:
a. A necessary operative procedure is chosen according to the etiology of the obstruction.
b. An obstruction is much more common in the jejunum than in the ileum at middle age !

c. Nonviable bowel should be resected.

d. Adhesions should be dissected


e. Tumor must be removed following ablastic principles.

The correct answer is: An obstruction is much more common in the jejunum than in the ileum at middle age

All of statements about Cloiberg's cup are correct EXCEPT

Select one:
a. Has a form of the inverted bowl
b. Can be observed after an enema
c. Cloiberg's cup is free air in an abdomen !

d. Sign of acute bowel obstruction

e. Is formed by the level of liquid and air

The correct answer is: Cloiberg's cup is free air in an abdomen

All of statements about high small bowel obstruction are correct EXCEPT

Select one:
a. Distension is prominent !

b. Colicky upper abdominal pain


c. Dehydration is present
d. Profuse, bilious vomiting

e. Vomitus is nonfeculent

The correct answer is: Distension is prominent

All of statements about large bowel obstruction are correct EXCEPT

Select one:
a. Colicky abdominal pain
b. Dehydration is present
c. Vomitus is feculent
d. Profuse, bilious vomiting !

e. Distension is prominent

The correct answer is: Profuse, bilious vomiting

An elderly patient is brought to the hospital with recent onset of colicky abdominal pain, distension and obstipation
on examination, the abdomen
is markedly distended and tympanitic. There is no marked tenderness. Plain abdominal x-ray shows a markedly
distended loop located mainly in the
right upper quadrant.
The likely diagnosis is:

Select one:
a. Sigmoid volvulus !

b. Small-bowel obstruction
c. Mesenteric vascular occlusion
d. Large-bowel obstruction
e. Gallstone ileus

The correct answer is: Sigmoid volvulus

Cloiberg's cup in large bowel obstruction is

Select one:
a. Narrow or wide and tall
b. Wide and low
c. Narrow and low
d. Narrow and tall !

e. Wide and tall

The correct answer is: Narrow and tall

Cloiberg's cup in small bowel obstruction is

Select one:
a. Narrow and tall
b. Narrow and low
c. Wide and tall
d. Wide and low !

e. Is located on each side of abdomen

The correct answer is: Wide and low

Comparing the anorectal manifestation of Crohn's dis¬ease and ulcerative colitis, which of the following is incorrect?

Select one:
a. Ulcerative colitis attacks always involve the rectum.
b. Perianal disease is common in Crohn's disease.
c. Bloody stools are less common in Crohn's disease than in ulcerative colitis.
d. Granulomas are a common finding on rectal biopsy in Crohn's disease.
e. Perirectal fistulas occur frequently in ulcerative colitis. !

The correct answer is: Perirectal fistulas occur frequently in ulcerative colitis.

Criteria suggesting a viability of intestine are all EXCEPT:

Select one:
a. Marginal arterial pulsations

b. Temperature of intestine
c. Absent of veins obstructed by clots. !

d. Peristalsis
e. Normal color

The correct answer is: Absent of veins obstructed by clots.

During surgery for bowel obstruction, cecum cancer was detected as a cause.The patient's condition is stable. No
tumor metastases
during abdominal revision were found. What kind of surgery is optimal?

Select one:
a. Tumor resection with primary anastomosis

b. Tumor biopsy
c. Right-sided hemicolectomy with primary anastomosis !

d. Сreating a bypass anastomosis


e. Ileostomy

The correct answer is: Right-sided hemicolectomy with primary anastomosis

During surgery for colonic obstruction, a sigmoid colon tumor was detected.The patient's condition is stable. The
transverse and descending colon is
expanded to 8 cm in diameter. No tumor metastases during abdominal revision were found. What kind of surgery is
optimal?

Select one:
a. Transversostomy

b. Сreating a bypass anastomosis


c. Left-sided hemicolectomy with transversostomy
d. Left-sided hemicolectomy with primary anastomosis
e. Hartmann's operation !

The correct answer is: Hartmann's operation

For a symptomatic partial duodenal obstruction secondary to an annular pancreas, the operative treatment of choice
is

Select one:
a. Vagotomy and gastrojejunostomy
b. Partial resection of the annular pancreas

c. Gastrojejunostomy
d. A Whipple procedure
e. Duodenojejunostomy !

The correct answer is: Duodenojejunostomy

Operative planning and preoperative counseling for a patient with a rectal carcinoma can be best provided if the
patient is staged before surgery by

Select one:
a. Barium enema
b. MRI of the pelvis

c. CT scanning of the pelvis


d. Rectal endosonography !

e. Rigid proctoscopy

The correct answer is: Rectal endosonography

The main reason of large bowel obstruction is

Select one:
a. Extrinsic inflammation
b. Adhesions

c. Volvulus
d. Hernias
e. Tumor !

The correct answer is: Tumor

The main reason of small bowel obstruction is

Select one:
a. Extrinsic tumor
b. Adhesions !

c. Volvulus

d. Extrinsic inflammation
e. Hernias

The correct answer is: Adhesions

The most informative investigation in acute bowel obstruction is

Select one:
a. Colonoscopy
b. Ultrasound
c. Barium enema

d. X-ray of abdomen !

e. Anoscopy

The correct answer is: X-ray of abdomen

The symptoms of bowel obstruction are all EXCEPT

Select one:
a. Obstipation.
b. Vomiting
c. Distention

d. Violet spots on face and trunk !

e. Abdominal pain

The correct answer is: Violet spots on face and trunk

Which of the following is not an indication for urgent surgery in patients with ulcerative colitis?

Select one:
a. Colonic perforation
b. Toxic megacolon
c. Ongoing hemorrhage
d. All are indications for urgent surgery

e. Failure of maximal medical management !

The correct answer is: Failure of maximal medical management

Which of the following would be expected to stimulate intestinal motility?

Select one:
a. Gastrin
b. Prolactin
c. Acetylcholine !

d. Secretin

e. Fear

The correct answer is: Acetylcholine

Finish review

◀︎ 04 Jump to... 05 ▶︎

Quiz navigation
1 2 3 4 5 6 7 8 9 10 11 12 13 14 15 16 17 18 19 20

Show one page at a time


Finish review

Copyright © 2022 Technomatix Ltd. | All Rights Reserved


( Distance Learning % &

" Dashboard Dashboard / My courses / 6 Surgery with pediatric surgery and oncology
/ 6 Syndrome of acute abdomen. Diagnosis, differential diagnosis and medical tactics. Differential diagnosis of acute
abdominal viscus diseases. Local and widespread inflammatory diseases of abdominal cavity organs and peritoneum.
! My Training Pseudoabdominal syndrome.
/ Test 6.1
" Catalog

# Calendar

$ Reports Started on Sunday, 25 September 2022, 12:47 PM


State Finished
Completed on Sunday, 25 September 2022, 12:55 PM
# 6 Surgery with pediatric
Time taken 7 mins 48 secs
surgery and oncology
Marks 20.00/20.00
$ Participants Grade 3.00 out of 3.00 (100%)

% Badges 55-year-old man who is extremely obese reports weakness, sweating, tachycardia, confusion, and headache
whenever he fasts for more than a few hours. He has prompt
& Competencies relief of symptoms when he eats. These symptoms are most suggestive of which of the following disorders?

' Grades Select one:


a. Carcinoid syndrome
b. Zollinger-Ellison syndrome
c. Insulinoma !

d. Diabetes mellitus
e. Multiple endocrine neoplasia

The correct answer is: Insulinoma

A 19-year-old college student returns from a trip to Cancun, and 2 weeks later develops malaise, weakness, and
anorexia. A week later he notices jaundice.
When he presents for evaluation his total bilirubin are 120, with 70 indirect and 50 direct. His alkaline phosphatase is
mildly elevated, and the SGOT and SGPT
(transaminases) are very high. What is it?

Select one:
a. Perforated duodenal ulcer
b. Acute mesenteric ischemia
c. Acute edematous pancreatitis

d. Hepatocellular jaundice !

e. Acute cholecystitis

The correct answer is: Hepatocellular jaundice

A 28-year-old previously healthy woman arrives in the emergency room complaining of 24 h of anorexia and nausea
and lower abdominal pain that is more intense
in the right lower quadrant than elsewhere. On examination she has peritoneal signs of the right lower quadrant and
a rectal temperature of 38.38C. At
exploration through incision of the right lower quadrant, she is found to have a small, contained perforation of a
cecal diverticulum. Which of the following
statements regarding this situation is true?

Select one:
a. Cecal diverticula are acquired disorders !

b. Diverticulectomy, closure of the cecal defect, and appendectomy may be indicated


c. An ileocolectomy is indicated even with well-localized inflammation
d. Cecal diverticula are usually multiple

e. Cecal diverticula are mucosal herniations through the muscularis propria

The correct answer is: Cecal diverticula are acquired disorders

A 32-year-old woman presents to the hospital with a 24-h history of abdominal pain of the right lower quadrant. She
undergoes an uncomplicated
appendectomy for acute appendicitis and is discharged home on the fourth postoperative day. The pathologist
notes the presence of a carcinoid
tumor (1.2 cm) in the tip of the appendix. Which of the following statements is true?

Select one:
a. The carcinoid syndrome occurs in more than half the patients with carcinoid tumors
b. The patient should be advised to undergo ileocolectomy
c. Carcinoid syndrome is seen only when the tumor is drained by the portal venous system
d. The tumor is an apudoma !

e. The most common location of carcinoids is in the appendix

The correct answer is: The tumor is an apudoma

A 33-year-old alcoholic man shows up in the ER with epigastria and midabdominal pain that began 12 hours ago
shortly after the ingestion of a large meal.
The pain is constant and very severe, and radiates straight through to the back. He vomited twice early on, but since
then has continued to have retching.
He has tenderness and some muscle guarding in the upper abdomen, is afebrile, and has mild tachycardia. Serum
amylase is 6, 5, and his hematocrit is 52%.
What is it?

Select one:
a. Acute mesenteric ischemia
b. Gallstones, with biliary colic
c. Perforated duodenal ulcer
d. Acute edematous pancreatitis !

e. Acute cholecystitis

The correct answer is: Acute edematous pancreatitis

A 43-year-old man develops excruciating abdominal pain at 8:18 PM. When seen in the ER at 8:50 PM, he has a rigid
abdomen, lies motionless on the
examining table, has no bowel sounds, and is obviously in great pain, which he describes as constant. X-ray shows
free air under the diaphragm. What is it?

Select one:
a. Acute cholangitis
b. Perforated duodenal ulcer !

c. Acute appendicitis
d. Acute pancreatitis
e. Acute diverticulitis

The correct answer is: Perforated duodenal ulcer

A 43-year-old obese mother of six children has severe right upper quadrant abdominal pain that began 6 hours ago.
The pain was colicky at first, radiated
to the right shoulder and around toward the back, and was accompanied by nausea and vomiting. For the past 2
hours the pain has been constant. She
has tenderness to deep palpation, muscle guarding, and rebound in the right upper quadrant. Her temperature is 101
°F, and she has a WBC count of 16,000.
She has had similar episodes of pain in the past brought about by ingestion of fatty food, but they all had been of
brief duration and relented spontaneously
or with anticholinergic medications. What is it?

Select one:
a. Acute mesenteric ischemia
b. Perforated duodenal ulcer
c. Acute appendicitis
d. Acute cholecystitis !

e. Acute pancreatitis

The correct answer is: Acute cholecystitis

A 59-year-old man arrives in the ER at 2 AM, accompanied by his wife who is wearing curlers on her hair and a robe
over her nightgown. He has abdominal pain
that began suddenly about 1 hour ago, and is now generalized, constant, and extremely severe. He lies motionless
on the stretcher, is diaphoretic, and has shallow,
rapid breathing. His abdomen is rigid, very tender to deep palpation, and has guarding and rebound tenderness in all
quadrants.

Select one:
a. Kidney colic
b. Cirrhosis of the liver
c. Right-side lobar pneumonia
d. Peritonism
e. Definitely an acute abdomen. !

The correct answer is: Definitely an acute abdomen.

A 59-year-old man is referred for evaluation because he has been fainting at his job where he operates heavy
machinery. He is pale and gaunt, but otherwise
his physical examination is remarkable only for 4+ occult blood in the stool. Lab shows a hemoglobin_of_50_g/l.
What is it?

Select one:
a. Acute cholecystitis
b. Perforated duodenal ulcer
c. Cancer of the right colon !

d. Acute appendicitis
e. Acute pancreatitis

The correct answer is: Cancer of the right colon

A 59-year-old woman has a history of three prior episodes of left lower quadrant abdominal pain for which she was
briefly hospitalized and treated with
antibiotics. She began to feel discomfort 12 hours ago, and now she has constant left lower quadrant pain,
tenderness, and a vaguely palpable mass. She has
fever and leukocytosis. What is it?

Select one:
a. Acute mesenteric ischemia

b. Acute diverticulitis !

c. Acute appendicitis
d. Perforated duodenal ulcer
e. Acute cholecystitis

The correct answer is: Acute diverticulitis

A 62-year-old man with cirrhosis of the liver and ascites presents with generalized abdominal pain that started 12
hours ago. He now has moderate tenderness over
the entire abdomen, with some guarding and equivocal rebound. He has mild fever and leukocytosis. What is it?

Select one:
a. Acute appendicitis

b. Perforated duodenal ulcer


c. Peritonitis in the cirrhotic with ascites !

d. Acute pancreatitis
e. Acute mesenteric ischemia

The correct answer is: Peritonitis in the cirrhotic with ascites

A 79-year-old man with atrial fibrillation develops an acute abdomen. He has a silent abdomen, with diffuse
tenderness and mild rebound. There is a trace
of blood in the rectal exam. He has acidosis and looks quite sick. X-rays show distended small bowel and distended
colon up to the middle of the transverse
colon. What is it?

Select one:
a. Perforated duodenal ulcer

b. Embolic occlusion of the mesenteric vessels !

c. Cancer of the right colon


d. Hepatocellular jaundice
e. Acute pancreatitis

The correct answer is: Embolic occlusion of the mesenteric vessels

A previously healthy 15-yearold boy is brought to the emergency room with complaints of about 12 h of progressive
anorexia, nausea, and pain of the right lower
quadrant. On physical examination, he is found to have a rectal temperature of 38.8C (and has direct and rebound
abdominal tenderness localizing to McBurney’s
point as well as involuntary guarding in the right lower quadrant. At operation through a McBurney-type incision, the
appendix and caecum are found to be normal,
but the surgeon is impressed with the marked edema of the terminal ileum, which also has an overlying fibrin
purulent exudates. The correct procedure is to

Select one:
a. Resects the involved terminal ileum
b. Perform the ileocolic resection

c. Close the abdomen after revision of the abdominal cavity, culturing the exudates and abdominal drainage !
d. Perform a standard appendectomy
e. Perform an ileocolostomy to bypass the involved terminal ileum

The correct answer is: Close the abdomen after revision of the abdominal cavity, culturing the exudates and
abdominal drainage

A white, obese 40-year-old mother of five children gives a history of repeated episodes of right upper quadrant
abdominal pain brought about by the ingestion
of fatty foods, and relieved by the administration of anticholinergic medications. The pain is colicky, radiates to the
right shoulder and around to the back,
and is accompanied by nausea and occasional vomiting. Physical examination is unremarkable. What is it?

Select one:
a. Gallstones, with biliary colic !

b. Perforated duodenal ulcer

c. Acute mesenteric ischemia


d. Acute pancreatitis
e. Cirrhosis of the liver

The correct answer is: Gallstones, with biliary colic

An 80-year-old man is admitted to the hospital complaining of nausea, abdominal pain, distention, and diarrhea. A
cautiously performed transanal contrast study
reveals an “apple core” configuration in the recto sigmoid. Appropriate management at this time would include

Select one:
a. Evaluation of an electrocardiogram and obtaining an angiogram to evaluate for colonic mesenteric ischemia
b. Colonoscopic decompression and rectal tube placement

c. Saline enemas and digital disimpaction of fecal matter from the rectum
d. Oral administration of metronidazole and checking a Clostridium difficile titer
e. Colon resection and proximal colostomy !

The correct answer is: Colon resection and proximal colostomy

An 88-year-old man with a history of end-stage renal failure, severe coronary artery disease, and brain metastases
from lung cancer presents with acute
cholecystitis. His family wants “everything done.” The best management option in this patient would be:

Select one:
a. Lithotripsy followed by long-term bile acid therapy
b. Laparoscopic cholecystectomy
c. Intravenous antibiotics followed by elective cholecystectomy

d. Open cholecystectomy
e. Tube cholecystostomy !

The correct answer is: Tube cholecystostomy

Which of the following statements regarding appendicitis during pregnancy is correct?

Select one:
a. Noncomplicated appendicitis results in a 20% fetal mortality and premature labor rate
b. Appendicitis is the most prevalent extra uterine indication for celiotomy during pregnancy !

c. Suspected appendicitis in a pregnant woman should be managed with a period of observation of due to the
risks of laparotomy to the fetus
d. Appendicitis occurs more commonly in pregnant women than in nonpregnant women of comparable age

e. The severity of appendicitis correlates with increased gestational age of the fetus

The correct answer is: Appendicitis is the most prevalent extra uterine indication for celiotomy during pregnancy

Which of the following statements regarding direct inguinal hernias is true?

Select one:
a. They commonly protrude into the scrotal sac in men
b. They incarcerate more commonly than indirect hernias
c. They should be opened and ligated at the internal ring
d. They protrude medially to the inferior epigastria vessels !

e. They are the most common inguinal hernias in women

The correct answer is: They protrude medially to the inferior epigastria vessels

Which of the following statements regarding the etiology of obstructive jaundice is true?

Select one:
a. A Klatskin tumor will result in extra hepatic ductal dilation only
b. A markedly elevated SGOT and SGPT are usually associated with obstructive jaundice
c. When extra hepatic biliary obstruction is suspected, the first test should be ultrasonography !

d. Carcinoma of the head of the pancreas can cause deep epigastria or back pain in as many as 80% of patients
e. A liver-spleen scan will add significantly to the diagnostic workup for obstructive jaundice

The correct answer is: When extra hepatic biliary obstruction is suspected, the first test should be ultrasonography

Which statement concerning cholangitis is correct?

Select one:
a. The disease occurs primarily in young, immunocompromised patients
b. Surgery is indicated once the diagnosis of cholangitis is made
c. The diagnosis is suggested by the Charcot triad
d. The most common infecting organism is Staphylococcus aureus
e. Cholecystostomy is the procedure of choice in affected patients !

The correct answer is: Cholecystostomy is the procedure of choice in affected patients

Finish review

◀︎ 05 Jump to... Test 6.2 ▶︎

Quiz navigation
1 2 3 4 5 6 7 8 9 10 11 12 13 14 15 16 17 18 19 20

Show one page at a time


Finish review

Copyright © 2022 Technomatix Ltd. | All Rights Reserved


( Distance Learning % &

" Dashboard Dashboard / My courses / 6 Surgery with pediatric surgery and oncology
/ 6 Syndrome of acute abdomen. Diagnosis, differential diagnosis and medical tactics. Differential diagnosis of acute
abdominal viscus diseases. Local and widespread inflammatory diseases of abdominal cavity organs and peritoneum.
! My Training Pseudoabdominal syndrome.
/ Test 6.2
" Catalog

# Calendar

$ Reports Started on Monday, 26 September 2022, 6:26 PM


State Finished
Completed on Monday, 26 September 2022, 6:30 PM
# 6 Surgery with pediatric
Time taken 4 mins 10 secs
surgery and oncology
Marks 20.00/20.00
$ Participants Grade 3.00 out of 3.00 (100%)

% Badges A 20-year-old man has undergone appendectomy for perforated appendicitis with generalized peritonitis. Seven
days postoperatively, his temperature continues to
& Competencies spike to 38, 8 0 C despite antibiotic therapy with ampicillin, gentamicin, and metronidazole. A CN scan reveals a
large pelvic abscess. Soon afterward, he has
' Grades bleeding from the surgical wound and all intravenous puncture sites. It is due to:

Select one:
a. Congenital bleeding disorder
b. Anaphylactoid reaction to intravenous dye
c. Liver failure

d. Disseminated intravascular coagulation


e. Antibiotic-induced coagulopathy !

The correct answer is: Antibiotic-induced coagulopathy

A 30-year-old man with a history of Crohn’s disease develops an enterocutaneous fistula and is placed on total
parenteral nutrition through a right subclavian
central venous catheter. After five days, the patient develops a fever and leukocytosis; CT scan of the abdomen
reveals no intraabdominal abscess. The subclavian
catheter insertion site is inspected and noted to be erythematous and painful. Blood cultures are positive. Which of
the following organisms is the most likely
cause of his fever?

Select one:
a. Coagulase-positive staphylococcus
b. Escherichia coli
c. Group A streptococcus

d. Enterococcus
e. Coagulase-negative staphylococci !

The correct answer is: Coagulase-negative staphylococci

A 38-year-old man with a history of fever associated with abdominal pain of 3 week’s duration presents now with a
sudden onset of explosive abdominal pain and
vomiting. Plain abdominal x-ray reveals air under a diaphragm. A CT scan shows mesenteric lymphadenopathy and
splenomegaly is found. Laparotomy is performed and
3 feet of ileum resected. The luminal aspect of the resected bowel shows marked ulceration of Peyer’s patches.
What is the most likely diagnosis?

Select one:
a. Crohn’s disease !

b. Tuberculosis enteritis
c. Typhoid enteritis
d. Primary peritonitis

e. Ulcerative colitis

The correct answer is: Crohn’s disease

A 45-year-old woman is explored for a perforated duodenal ulcer 6 h after onset of symptoms. She has a history of
chronic peptic ulcer disease treated medically with minimal symptoms. The procedure of choice is:

Select one:
a. Excision of the ulcer, duodenoplasty !

b. Antrectomy and truncal vagotomy


c. Highly selective vagotomy
d. Simple closure with omental patch

e. Hemigastrectomy

The correct answer is: Excision of the ulcer, duodenoplasty

A 70-year-old man with aortic and mitral valvular regurgitation undergoes an emergency sigmoid colectomy and end
colostomy for perforated diverticulitis.
His postoperative course is complicated by a myocardial infarction and atrial fibrillation. Four weeks later, he has
improved and requests elective colostomy closure.
You would recommend:

Select one:
a. Control of congestive heart failure with diuretics and digitalis in severe cases !

b. Discontinuation of antiarrhythmic and antihypertensive medications on the morning of surgery


c. Administration of prophylactic antibiotics, other than ampicillin and gentamicin, for patients with valvular
heart disease who are undergoing gastrointestinal procedures
d. Discontinuation of beta-blocking medications on the day prior to surgery
e. Postponement of elective surgery for 6–8 wk after a subendocardial myocardial infarction

The correct answer is: Control of congestive heart failure with diuretics and digitalis in severe cases

All of the following statements concerning intraabdominal abscesses are correct EXCEPT:

Select one:
a. Treatment usually includes surgical exploration with drainage of the abscess
b. A common cause is perforation of hollow viscus
c. Ultrasonography, CT scan, or MRI can be used to direct surgical drainage
d. These infections are usually due to staphylococcal organism !

e. A high index of suspicious is essential for the diagnosis because there may be no physical signs of infection

The correct answer is: These infections are usually due to staphylococcal organism

An elderly diabetic woman with chronic steroid-dependent bronchospasm has an ileocolectomy for a perforated
cecum. She is taken to the ICU intubated and is
maintained on broad-spectrum antibiotics, renal-dose dopamine, and a rapid steroid taper. On postoperative day 2
she develops a fever of 39.2°C (102.5°F),
hypotension, lethargy, and laboratory values remarkable for hypoglycemia and hyperkalemia. The most likely
diagnosis of this acute event is:

Select one:
a. Diabetic ketoacidosis

b. Adrenal insufficiency !

c. Sepsis
d. Acute tubular necrosis
e. Hypovolemia

The correct answer is: Adrenal insufficiency

Choose cytokine which role in the development of the systemic inflammatory response syndrome and multiple organ
failure is proved:

Select one:
a. TNF-α

b. IL-2
c. IL-10
d. CRP
e. leptin !

The correct answer is: leptin

Choose the incorrect statement

Select one:
a. As the fibrin matrix matures, the bacteria within are protected from host clearance mechanisms

b. Production of fibrinous exudates is considered an important part of the host defense


c. Peritonitis causes a reduction in the intra-abdominal fibrinolytic activity
d. Fibrinolytics improve the outcome !

e. Large numbers of bacteria may be sequestered within the fibrin matrix

The correct answer is: Fibrinolytics improve the outcome

Following factors lead to persistence of infections and abscess formation EXCEPT:

Select one:
a. Facultative anaerobic growth
b. Adhesion capabilities

c. Capsule formation
d. Increase in the intraabdominal fibrinolytic activity
e. Succinic acid production !

The correct answer is: Succinic acid production

Laparoscopy in abdominal trauma may be indicated in which of the following?

Select one:
a. In hemodynamically unstable patients !

b. If there is limited cardiovascular reserve

c. In patients with multiple previous abdominal operations


d. If severe diffuse peritonitis exist
e. To exclude diaphragmatic injury

The correct answer is: In hemodynamically unstable patients

Postoperatively, the patient requires an indwelling bladder catheter for 5 days to treat urinary retention. He does well
until the tenth postoperative day,
at which point he develops a fever of 39 C0, right lower quadrant pain, and an ileus. The midline wound is not
inflamed. The most likely development is:

Select one:
a. Pyelonephritis
b. Pseudomembranous enterocolitis

c. Blind loop syndrome


d. Recurrent Crohn’s disease
e. Intraabdominal abscess !

The correct answer is: Intraabdominal abscess

Predisposing factors for the development of abdominal candidiasis include the following EXCEPT:

Select one:
a. Laparostomy !

b. Gastric acid suppressive therapy


c. Prolonged use of broad-spectrum antibiotics

d. Central venous catheters and intravenous hyperalimentation


e. Steroids and other forms of immunosuppression

The correct answer is: Laparostomy

Temporary closure of the abdomen to prevent herniation and contamination from the outside of the abdominal
contents can be achieved using:

Select one:
a. Adhesive tape
b. Velcro-like closure devices
c. Mesh

d. Self-adhesive membrane dressings


e. Vacuum-assisted closure (VAC) devices !

The correct answer is: Vacuum-assisted closure (VAC) devices

The 19-year-old female is brought to the emergency room after being in a single-car accident just 20 min earlier in
which she lost control of her car and hit
a retaining column of an overpass at about 45 miles per hour. She was the only occupant and was wearing a seat
belt but looks pale, has tachycardia and
positional hypotension, is extremely nauseated, and is lying in the fetal position due to severe abdominal pain. She
does not appear to have any broken bones and
a cranial nerve test appears normal. You order an abdominal CT because you suspect which of the following?

Select one:
a. That she has peritonitis from a ruptured gallbladder
b. That she is pregnant
c. That she has hemorrhoids

d. That she has diverticulitis


e. That she has peritonitis from a ruptured spleen !

The correct answer is: That she has peritonitis from a ruptured spleen

The clinical signs of peritonitis does not include:

Select one:
a. Jugular veins distension !

b. Anorexia and nausea


c. Tenderness to palpation of the abdomen
d. Increased abdominal wall rigidity

e. Abdominal pain

The correct answer is: Jugular veins distension

The general principles guiding the treatment of intra-abdominal infections include the following EXCEPT:

Select one:
a. To control the inflammatory process
b. To eliminate bacteria and toxins
c. To prescribe corticosteroids !

d. To maintain organ system function

e. To control the infectious source

The correct answer is: To prescribe corticosteroids

Which of the following statements regarding appendicitis during pregnancy is correct?

Select one:
a. Suspected appendicitis in a pregnant woman should be managed with a period of observation of due to the
risks of laparotomy to the fetus
b. Noncomplicated appendicitis results in a 20% fetal mortality and premature labor rate
c. Appendicitis is the most prevalent extrauterine indication for celiotomy during pregnancy !

d. Appendicitis occurs more commonly in pregnant women than in nonpregnant women of comparable age
e. The severity of appendicitis correlates with increased gestational age of the fetus

The correct answer is: Appendicitis is the most prevalent extrauterine indication for celiotomy during pregnancy

Which one of the following cases is considered a clean-contaminated wound?

Select one:
a. Appendectomy with walled-off abscess
b. Lumpectomy with axillary node dissection
c. Open cholecystectomy for cholelithiasis !

d. Herniorrhaphy with mesh repair


e. Gunshot wound to the abdomen with injuries to the small bowel and sigmoid colon

The correct answer is: Open cholecystectomy for cholelithiasis

While treating SBP should be avoided:

Select one:
a. Metronidazole

b. Quinolone
c. Cephalosporins
d. Norfloxacin
e. Aminoglycosides !

The correct answer is: Aminoglycosides

Finish review

◀︎ Test 6.1 Jump to... 06.1 ▶︎

Quiz navigation
1 2 3 4 5 6 7 8 9 10 11 12 13 14 15 16 17 18 19 20

Show one page at a time


Finish review

Copyright © 2022 Technomatix Ltd. | All Rights Reserved


( Distance Learning % &

" Dashboard Dashboard / My courses / 6 Surgery with pediatric surgery and oncology
/ 7 Mechanical jaundice. Causes. Differential diagnostic tactics. Modern approaches to treatment. Acute hepatic
insufficiency in case of surgical diseases, methods of treatment and prophylaxis.
! My Training / Тест 7

" Catalog

# Calendar
Started on Sunday, 2 October 2022, 9:21 AM
$ Reports State Finished
Completed on Sunday, 2 October 2022, 9:23 AM
Time taken 2 mins 28 secs
# 6 Surgery with pediatric
surgery and oncology Marks 20.00/20.00
Grade 3.00 out of 3.00 (100%)
$ Participants
43 years old patient complains of having night pain in the right subcostal area, general weakness, bloating, and
% Badges yellowness of the scleras. ALT 1.9., AST 2.4., bilirubin 88 µmol/L (direct 40, indirect 48). What is your
recommendation for treatment?
& Competencies
Select one:
' Grades a. Bile stimulated. !

b. H2-blockers.
c. Antispasmodic.

d. Ice on the abdomen wall.


e. Vitamins.

The correct answer is: Bile stimulated.

60 years old patient, male, has jaundice for over 3 weeks, began without pain and pain is getting more intensive.
Abdomen palpation is soft. Symptom Courvoisier's is positive. Ultrasound marked biliary tract and gallbladder
enlargement choledoch. What is the origin of these changes?

Select one:
a. Chronic pancreatitis.
b. Calculous biliary disease.
c. Cancer head of the pancreas. !

d. Cancer of the liver.


e. Hepatitis.

The correct answer is: Cancer head of the pancreas.

A 48 old-year patient, male, sick for two weeks when he noticed icterus of the sclera. He notes on slight pain in the
right subcostal area. The pain is not acute. After two days of onset, appeared gray stools, jaundice of skin which
gradually increased. General condition is satisfactory. On sonography revealed enlargement intrahepatic and
extrahepatic bile ducts and main pancreatic duct. What is the most likely diagnosis?

Select one:
a. Total liver tumor.
b. Major duodenal papilla tumor. !

c. Norm cholestatic hepatitis.


d. Cirrhosis.

e. Choledocholithiasis.

The correct answer is: Major duodenal papilla tumor.

A male patient, 62 years old, complains of presence of jaundice, itching of skin, light stool and dark urine.
OBJECTIVE: scleras and skin are yellow, the traces of wound on skin. Gallbladder is enlarged and painless. Blood
tests: Total bilirubin 85 µmol / L. What method of research is optimally used to clarify the diagnosis?

Select one:
a. X-ray examination of the abdomen.
b. Duodenal-sensing.
c. Infusion cholegraphy.
d. Retrograde cholangiopancreatography. !

e. Oral-cholecystography.

The correct answer is: Retrograde cholangiopancreatography.

Child 9-years old complaints of abdominal pain, more from the right, subfebrile temperature, vomiting that does not
bring relief. In the blood, clotting speed of erythrocytes was found. He has been already ill for three years. What
disease caused this clinical picture?

Select one:
a. Dyskinesia of the bill duct.
b. Ulcerative colitis
c. Acute intestinal infection.
d. Chronic cholecystitis !

e. Chronic enterocolitis

The correct answer is: Chronic cholecystitis

Complains of acute cholangitis are:

Select one:
a. Labile and liquid stools.
b. Ground.
c. Zoster pain.
d. High temperature, pain in the right subcostal area, jaundice. !

e. Nausea, vomiting.

The correct answer is: High temperature, pain in the right subcostal area, jaundice.

During operation with acute destructive cholecystitis presence of purulent cholangitis. While there were no
choledoch calculus and duodenal stenosis, symptom nipple was present. Gallbladder is removed. How to finish the
operation?

Select one:
a. Supraduodenal choledochoduodenoanastomois.
b. Choledochojejunoanastomosis.
c. External drainage of the CBD. !

d. Drainage of abdominal cavity.


e. Transduodenal choledochoduodenostomy.

The correct answer is: External drainage of the CBD.

Female 53-year old came to the surgery department complaining of pain in the right subcostal area that radiates to
the right shoulder nausea, vomiting, fever 37.8 C. Her belly is swollen and stiff during palpation, tenderness in the
right subcostal, determined in the same dense, painful movements. Positive symptoms by Orthner and Mussy-
Georgievsky are positive. In the blood - leukocytes are 12.6·109/l. Formulate a preliminary diagnosis:

Select one:
a. Calculous biliary disease. !

b. Acute pancreatitis.
c. Perforated ulcer.
d. Acute gastroduodenitis.
e. Tumor of the liver

The correct answer is: Calculous biliary disease.

Female 53-year old was operated with cholecystectomy for acute calculous cholecystitis one month ago. The
operation finished outer drainage of choledoch. Cholangiography was not made. Drainage removed on 7 days after
the operation. Three days ago, the temperature increased to 38.8, had chills and sweating. Scleras are yellow, stool
is grey color. Abdominal palpation reveals pain in the right subcostal area. Formulate a preliminary diagnosis?

Select one:
a. Residual choledocholithiasis, cholangitis, obstructive jaundice. !

b. Abscess of the liver.


c. Postcholecystectomy syndrome.
d. Stenosis papillitis.
e. Stricture of the choledoch.

The correct answer is: Residual choledocholithiasis, cholangitis, obstructive jaundice.

Female 68-years old complaining on yellowness of skin and sclera, grey stool, dark colour of the urine. Body
temperature increased to 37.4 degrees. Jaundice appeared a day after attack of pain in the right subcostal area.
Abdomen soft, painful in the right subcostal area, liver is not increased. Symptom Murphy is positive. The
gallbladder is increased. What is the most probable diagnosis?

Select one:
a. Acute cholecystitis, obstructive jaundice. !

b. Cholestatic hepatitis.
c. Cholecystolythiasis.
d. Residual choledocholithiasis.

The correct answer is: Acute cholecystitis, obstructive jaundice.

Indications to choledochotomy:

Select one:
a. Acute pancreatitis.
b. Cholangitis, obstructive jaundice, stones in the duct. !

c. Perforated ulcer.
d. Hepatitis.
e. Empyema of the gallbladder.

The correct answer is: Cholangitis, obstructive jaundice, stones in the duct.

Patient R., female 63 year old, complaining of severe acute pain in the right half of the abdomen, which irradiates to
the right supraclavicular area, an increase in body temperature, dryness and bitterness in the mouth. There have
been repeated vomiting, which does not bring relief. The beginning of pain is linked with consumption of fatty and
fried foods. The patient lies on the right side. Pale. Tachycardia. Tongue is dry. Painful palpation in the right half area
of the abdomen and somewhat strained in the right subcostal area. Give probable diagnosis.

Select one:
a. Acute cholecystitis. !

b. Perforated ulcer of the stomach.

c. Acute appendicitis.
d. Right renal colic.
e. Acute intestinal obstruction.

The correct answer is: Acute cholecystitis.

The 38 old-years patient, after a cholecystectomy two years ago, had attacks of hepatic colic with yellowing of the
skin. Postcholecystectomy syndrome is diagnosed. Which of the following is often the cause of relapse?

Select one:
a. Duodenostasis.
b. Cholestasia.
c. Stenos papilitis.

d. Dyskinesia of the biliary duct.


e. Abandoned during the first operation calculus. !

The correct answer is: Abandoned during the first operation calculus.

The female patient 32 year-old, had a cholecystectomy two years ago, connected with acute calculous
cholecystitis.. In gallbladder calculus was much smaller. During six months, she has three times repeated attacks of
hepatic colic. Two days ago, after the attack, appeared yellowing of the skin and sclera. Bilirubin is 90 µmol /L. What
research can help to diagnose this patient effectively?

Select one:
a. Ray examination of gastrointestinal tract.
b. Oral-cholangiography.
c. Gastroduodenoscopy.

d. Angiography.
e. Ultrasound of pancreatoduodenal zone. !

The correct answer is: Ultrasound of pancreatoduodenal zone.

The female patient operated on of chronic calculous cholecystitis and choledocholithiasis. Three years after surgery
patient has a recurrent choledocholithiasis. Which of the listed causes of non-core, leading to relapse in
extrahepatic bile duct calculus after a cholecystectomy?

Select one:
a. In-seam threads education CBD not resolves.
b. Narrowing of the terminal part of the CBD and associated cholangitis.
c. Availability impede outflow of bile.

d. Failure to comply with the recommendations of diet. !

e. Long-cystic duct stump.

The correct answer is: Failure to comply with the recommendations of diet.

The patient 32 year-old, female, after cholecystectomy 6 months ago, regenerates a liver colic attacks accompanied
yellowing of skin. On ultrasonography revealed left 2 x 1 cm calculus in distal part of choledoch. What treatment is
best for the patient?

Select one:
a. Conservative therapy and antispasmodic drugs.
b. Choledochoduodenoanastomosis.
c. Endoscopic + duodenocholedochoscopy with papilosphincterotomy. !

d. Lithotripsy.

e. Choledocholithotomy.

The correct answer is: Endoscopic + duodenocholedochoscopy with papilosphincterotomy.

The patient 52 year-age after the attack of pain in right subcostal area, had yellowness of skin and mucous
membranes. On ultrasound investigation, calculi were found in the gallbladder, choledoch was extended to 1.5 cm,
and choledoch content is not clearly visualized. Which test will confirm of choledocholithiasis?

Select one:
a. Endoscopic retrograde сholangiography. !

b. Blood tests for bilirubin and liver test.


c. Ultrasound re-examination after training.
d. Computed tomography.

e. Fibrogastroduodenosсopy.

The correct answer is: Endoscopic retrograde сholangiography.

What operation results in syndrome of “blend bag”?

Select one:
a. External drainage of choledochal.
b. Papillosphincterotomy.
c. Choledochoduodenostomy. !

d. Cholecystectomy.

e. Choledochotomy.

The correct answer is: Choledochoduodenostomy.

What type of operation is used for removing stones in ampoule of the major duodenal papilla?

Select one:
a. Cholecystectomy.
b. Choledochotomy.
c. External drainage of choledochal.
d. Transduodenal papillosphincterotomy. !

e. Segmental resection of choledochal.

The correct answer is: Transduodenal papillosphincterotomy.

Which of these operations used for postcholecystectomy syndrome and stenosis of large duodenal papilla?

Select one:
a. Endoscopic papillosphincterotomy. !

b. Choledochojejunoanastomosis.
c. Resection of the major duodenal nipple.
d. Transduodenal choledochoduodenostomy.
e. Hepaticotomy.

The correct answer is: Endoscopic papillosphincterotomy.

Finish review

◀︎ 06.2 Jump to... 07 ▶︎

Quiz navigation
1 2 3 4 5 6 7 8 9 10 11 12 13 14 15 16 17 18 19 20

Show one page at a time


Finish review

Copyright © 2022 Technomatix Ltd. | All Rights Reserved


! Distance Learning ! "

Dashboard / My courses / 6 Surgery with pediatric surgery and oncology


/ 8 Modern principles of surgical treatment of complicated peptic ulcer of the stomach and duodenum. / Test 8

Started on Monday, 3 October 2022, 6:36 PM


State Finished
Completed on Sunday, 9 October 2022, 8:45 PM
Time taken 6 days 2 hours
Marks 18.00/40.00
Grade 1.35 out of 3.00 (45%)

A 72-year-old patient with an intractable type I ulcer along the incisura with a significant amount of scarring along the entire length of the lesser curvature.
Select the appropriate surgical procedure for each patient.

a. vagotomy and antrectomy


b. antrectomy alone
c. vagotomy and gastrojejunostomy

d. vagotomy and pyloroplasty


e. proximal gastric vagotomy

Omeprazole has been added to the H2-antagonists as a therapeutic approach to the management of acute gastric and duodenal ulcers. It acts by

a. blocking breakdown of mucosal-damaging metabolites of NSAIDs


b. providing a direct cytoprotective effect
c. inhibiting parietal cell hydrogen-potassium-ATPase

d. buffering gastric acids


e. inhibiting gastrin release and parietal cell acid production

Intragastric pressure remains steady near 25 mm Hg during slow gastric filling, but rises rapidly to high levels after reaching a volume of

a. 700-900 mL
b. 1300-1500 mL
c. 400-600 mL
d. 1000-1200 mL

e. 1600-1800 mL

A 55-year-old man who is extremely obese reports weakness, sweating, tachycardia, confusion, and headache whenever he fasts for more than a few hours.
He has prompt relief of symptoms when he eats. These symptoms are most suggestive of which of the following disorders?

a. carcinoid syndrome
b. diabetes mellitus
c. insulinoma
d. Zollinger-Ellison syndrome

e. multiple endocrine neoplasia, type II

What substance is secreted by the C cells?

a. Gastric acid
b. Glucagon
c. Pepsinogen
d. Gastrin

e. Pepsin

A 42-year-old man with no history of use of nonsteroidal antiinflammatory drugs (NSAIDs) presents with recurrent gastritis. Infection with Helicobacter pylori
is suspected. Which of the following statements is true?

a. The organism is easily eradicated


b. Morphologically, the bacteria is a gram-positive, tennis-racket-shaped organism
c. Diagnosis can be made by serologic testing or urea breath tests
d. The most effective way to treat and prevent recurrence of this patients gastritis is through the use of single-drug therapy aimed at eradicating H. pylori
e. Diagnosis is most routinely achieved via culturing endoscopic scrapings

A 30-year-old man with a duodenal ulcer is being considered for surgery. His serum gastrin level, however, is 150+10 pg/mL on three determinations. The
surgeon should perform

a. An arteriogram
b. A secretin stimulation test
c. A total gastrectomy
d. A highly selective vagotomy
e. A subtotal gastrectomy

A 72-year-old patient with an intractable type I ulcer along the incisura with a significant amount of scarring along the entire length of the lesser curvature.
Select the appropriate surgical procedure for each patient.

a. Antrectomy alone
b. Vagotomy and gastrojejunostomy
c. Vagotomy and pyloroplasty
d. Proximal gastric vagotomy
e. Vagotomy and antrectomy

A 30-year-old man with a duodenal ulcer is being considered for surgery because of intractable pain and a previous bleeding episode. Serum gastrin levels
are found to be over 1000 pg/mL (normal 40-150) on three separate determinations. The patient should be told that the operation of choice is

a. Partial pancreatectomy
b. Subtotal gastrectomy
c. Highly selective vagotomy and tumor resection
d. Vagotomy and pyloroplasty

The typical carcinoma that develops in association with Barrett’s esophagus is

a. squamous cell
b. small cell

c. epidermoid
d. mucoepidermoid
e. adenocarcinoma

Local stimuli that inhibit the release of gastrin from the gastric mucosa include which of the following?

a. 20-proof alcohol
b. acidic antral contents

c. caffeine
d. small proteins
e. antral distention

The blood supply to the stomach and duodenum arises from all of the following arteries EXCEPT

a. Splenic artery
b. Inferior mesenteric artery

c. Superior mesenteric artery


d. Gastroepiploic artery
e. Common hepatic artery

A 42-year-old man with no history of use of nonsteroidal antiinflammatory drugs (NSAIDs) presents with recurrent gastritis. Infection with Helicobacter pylori
is suspected. Which of the following statements is true?

a. the most effective way to treat and prevent recurrence of this patients gastritis is through the use of single-drug therapy aimed at eradicating H. pylori
b. diagnosis can be made by serologic testing or urea breath tests
c. the organism is easily eradicated

d. morphologically, the bacteria is a gram-positive, tennis-racket-shaped organism


e. diagnosis is most routinely achieved via culturing endoscopic scrapings

Six weeks after surgery, the patient returns complaining of postprandial weakness, sweating, light-headedness, crampy abdominal pain, and diarrhea. The
best management would be

a. dietary advice and counseling that symptoms will probably abate within 3 mo of surgery
b. dietary advice and counseling that symptoms will probably not abate but are not dangerous
c. workup for neuroendocrine tumor (e.g., carcinoid)

d. preparation for revision to Roux-en-Y gastrojejunostomy


e. antispasmodic medications (e.g., Lomotil)

A 55-year-old man who is extremely obese reports weakness, sweating, tachycardia, confusion, and headache whenever he fasts for more than a few hours.
He has prompt relief of symptoms when he eats. These symptoms are most suggestive of which of the following disorders?

a. Zollinger-Ellison syndrome
b. Multiple endocrine neoplasia, type II
c. Insulinoma

d. Diabetes mellitus
e. Carcinoid syndrome

Six weeks after surgery, the patient returns complaining of postprandial weakness, sweating, light-headedness, crampy abdominal pain, and diarrhea. The
best management would be

a. Dietary advice and counseling that symptoms will probably not abate but are not dangerous
b. Preparation for revision to Roux-en-Y gastrojejunostomy
c. Dietary advice and counseling that symptoms will probably abate within 3 mo of surgery
d. Antispasmodic medications (e.g., Lomotil)

e. Workup for neuroendocrine tumor (e.g., carcinoid)

Gastroesophageal reflux is best characterized by which of the following statements?

a. It may be associated with increased gastrin production


b. It is diagnosed by manometry and 24-hour monitoring of pH in the lower esophagus
c. It is a relative contraindication to esophagoscopy
d. It is synonymous with hiatal hernia

e. It results from a higher than normal lower esophageal sphincter pressure

Intragastric pressure remains steady near 25 mm Hg during slow gastric filling, but rises rapidly to high levels after reaching a volume of

a. 1300-1500 mL
b. 1600-1800 mL
c. 700-900 mL
d. 400-600 mL

e. 1000-1200 mL

Which of the following statements regarding stress ulceration is true?

a. it is true ulceration, extending into and through the muscularis mucosa


b. increased secretion of gastric acid has been shown to play a causative role
c. it is seen following shock or sepsis, but for some unknown reason does not occur following major surgery, trauma, or burns
d. it classically involves the antrum
e. it frequently involves multiple sites

The blood supply to the stomach and duodenum arises from all of the following arteries EXCEPT

a. gastroepiploic artery
b. superior mesenteric artery
c. inferior mesenteric artery
d. splenic artery
e. common hepatic artery

Gastroesophageal reflux is best characterized by which of the following statements?

a. it is a relative contraindication to esophagoscopy


b. it is diagnosed by manometry and 24-hour monitoring of pH in the lower esophagus
c. it results from a higher than normal lower esophageal sphincter pressure
d. it may be associated with increased gastrin production
e. it is synonymous with hiatal hernia

A 45-year-old woman is explored for a perforated duodenal ulcer 6 h after onset of symptoms. She has a history of chronic peptic ulcer disease treated
medically with minimal symptoms. The procedure of choice is

a. Truncal vagotomy and pyloroplasty


b. Antrectomy and truncal vagotomy
c. Highly selective vagotomy
d. Simple closure with omental patch
e. Hemigastrectomy

Local stimuli that inhibit the release of gastrin from the gastric mucosa include which of the following?

a. Antral distention

b. Acidic antral contents


c. Caffeine
d. 20-proof alcohol
e. Small proteins

Which of the following organisms is most closely associated with gastric and duodenal ulcer disease?

a. Campylobacter

b. Mycobacterium avium-intracellulare
c. Cytomegalovirus
d. Yersinia enterocolitica
e. Helicobacter

A 45-year-old woman is explored for a perforated duodenal ulcer 6 h after onset of symptoms. She has a history of chronic peptic ulcer disease treated
medically with minimal symptoms. The procedure of choice is

a. highly selective vagotomy

b. simple closure with omental patch


c. antrectomy and truncal vagotomy
d. truncal vagotomy and pyloroplasty
e. hemigastrectomy

Which vessel is most commonly associated with a posterior duodenal ulcer?

a. Middle colic artery


b. Gastroduodenal artery

c. Superior mesenteric artery


d. Right gastroepiploic artery
e. Common hepatic artery

A 30-year-old man with a duodenal ulcer is being considered for surgery because of intractable pain and a previous bleeding episode. Serum gastrin levels
are found to be over 1000 pg/mL (normal 40-150) on three separate determinations. The patient should be told that the operation of choice is

a. vagotomy and pyloroplasty


b. total gastrectomy

c. highly selective vagotomy and tumor resection


d. partial pancreatectomy
e. subtotal gastrectomy

Gastric acid production is altered by all of the following hormones or actions EXCEPT

a. Cholecystokinin
b. Secretin

c. Gastrin
d. Vagal stimulation
e. Glucagon

Omeprazole has been added to the H2-antagonists as a therapeutic approach to the management of acute gastric and duodenal ulcers. It acts by

a. Inhibiting gastrin release and parietal cell acid production


b. Providing a direct cytoprotective effect
c. Buffering gastric acids

d. Blocking breakdown of mucosal-damaging metabolites of NSAIDs


e. Inhibiting parietal cell hydrogen-potassium-ATPase

The lower esophageal sphincter pressure is increased by

a. chocolate
b. astrin
c. emptying of the stomach

d. acid in the stomach


e. glucagon

A 55-year-old man complains of chronic intermittent epigastric pain, and gastroscopy demonstrates a 2-cm ulcer of the distal lesser curvature. Endoscopic
biopsy yields no malignant tissue. After a 6-wk trial of H2-blockade and antacid therapy, the ulcer is unchanged. Proper therapy at this point is

a. repeat trial of medical therapy


b. vagotomy and pyloroplasty
c. local excision of the ulcer

d. Billroth I partial gastrectomy


e. Billroth I partial gastrectomy with vagotomy

What substance is secreted by the C cells?

a. gastrin
b. pepsinogen
c. glucagon
d. pepsin

e. gastric acid

Which vessel is most commonly associated with a posterior duodenal ulcer?

a. superior mesenteric artery


b. common hepatic artery
c. gastroduodenal artery
d. middle colic artery

e. right gastroepiploic artery

The lower esophageal sphincter pressure is increased by

a. Acid in the stomach


b. Chocolate
c. Emptying of the stomach
d. Glucagon

e. Gastrin

A 55-year-old man complains of chronic intermittent epigastric pain, and gastroscopy demonstrates a 2-cm ulcer of the distal lesser curvature. Endoscopic
biopsy yields no malignant tissue. After a 6-wk trial of H2-blockade and antacid therapy, the ulcer is unchanged. Proper therapy at this point is

a. Billroth I partial gastrectomy


b. Billroth I partial gastrectomy with vagotomy
c. Vagotomy and pyloroplasty
d. Local excision of the ulcer
e. Repeat trial of medical therapy

A 30-year-old man with a duodenal ulcer is being considered for surgery. His serum gastrin level, however, is 150+10 pg/mL on three determinations. The
surgeon should perform

a. a total gastrectomy
b. a highly selective vagotomy
c. an arteriogram
d. a secretin stimulation test
e. a subtotal gastrectomy

Gastric acid production is altered by all of the following hormones or actions EXCEPT

a. secretin
b. cholecystokinin
c. glucagon
d. gastrin
e. vagal stimulation

Which of the following statements regarding stress ulceration is true?

a. It is true ulceration, extending into and through the muscularis mucosa


b. It is seen following shock or sepsis, but for some unknown reason does not occur following major surgery, trauma, or burns
c. It frequently involves multiple sites
d. It classically involves the antrum
e. Increased secretion of gastric acid has been shown to play a causative role

Which of the following organisms is most closely associated with gastric and duodenal ulcer disease?

a. Yersinia enterocolitica

b. Campylobacter
c. Mycobacterium avium-intracellulare
d. Cytomegalovirus
e. Helicobacter

The typical carcinoma that develops in association with Barrett’s esophagus is

a. Squamous cell

b. Epidermoid
c. Adenocarcinoma
d. Mucoepidermoid
e. Small cell

Finish review

◀︎ 07 Jump to... 08.1 ▶︎

Quiz navigation
1 2 3 4 5 6 7 8 9 10 11 12 13 14 15 16 17 18 19 20 21 22 23 24 25 26 27 28 29 30 31

32 33 34 35 36 37 38 39 40

Show one page at a time


Finish review

Copyright © 2022 Technomatix Ltd. | All Rights Reserved


( Distance Learning % &

" Dashboard Dashboard / My courses / 6 Surgery with pediatric surgery and oncology
/ 9 Bleeding from the digestive tract. Causes of origin, diagnosis and differential diagnosis, treatment tactic. / Тест 9

! My Training

" Catalog
Started on Saturday, 15 October 2022, 10:42 AM
# Calendar State Finished
Completed on Saturday, 15 October 2022, 10:45 AM
$ Reports
Time taken 3 mins 39 secs
Marks 20.00/20.00
# 6 Surgery with pediatric Grade 3.00 out of 3.00 (100%)
surgery and oncology
A 30-year-old man with a duodenal ulcer is being considered for surgery because of intractable pain and a previous
$ Participants bleeding episode. Serum gastrin levels are found to be over 1000 pg/mL (normal 40-150) on three separate
determinations. The patient should be told that the operation of choice is:
% Badges
Select one:
& Competencies a. Highly selective vagotomy and tumor resection. !

b. Subtotal gastrectomy.
' Grades
c. Vagotomy and pyloroplasty.

d. Total gastroctomy.
e. Partial pancreatectomy.

The correct answer is: Highly selective vagotomy and tumor resection.

A 50-year-old man is admitted with massive, bright red rectal bleeding. He recently had a barium enema that
demonstrated no diverticular or space-occupying lesion. Nasogastric suction reveals no blood but does produce
yellow bile. The patient continues to bleed. What is the next diagnostic step?

Select one:
a. Mesenteric angiography. !

b. Colonoscopy.
c. Repeat barium enema.

d. Small bowel follow-through with barium.


e. Upper GI series.

The correct answer is: Mesenteric angiography.

A 55-year-old man complains of chronic intermittent epigastric pain, and gastroscopy demonstrates a 2-cm ulcer of
the distal lesser curvature. Endoscopic biopsy yields no malignant tissue. After a 6-wk trial of H2-blockade and
antacid therapy, the ulcer is unchanged. Proper therapy at this point is:

Select one:
a. Local excision of the ulcer.
b. Billroth I partial gastrectomy with vagotomy.
c. Vagotomy and pyloroplasty.
d. Billroth I partial gastrectomy. !

e. Repeat trial of medical therapy.

The correct answer is: Billroth I partial gastrectomy.

A 55-year-old man complains of chronic intermittent epigastric pain, and gastroscopy demonstrates a 2-cm ulcer of
the distal lesser curvature. Endoscopic biopsy yields no malignant tissue. After a 6-wk trial of H2-blockade and
antacid therapy, the ulcer is unchanged. After initial resuscitation, this man should undergo:

Select one:
a. Selective angiography.
b. Exploratory celiotomy.
c. Barium swallow.
d. Esophagogastroscopy. !

e. Esophageal balloon tamponade.

The correct answer is: Esophagogastroscopy.

A 56-year-old man complains of the onset of severe substernal pain after a night of heavy drinking followed by
uncontrolled retching. He states that there was a small amount of blood in his vomit. A chest X-ray shows a
moderate sized left pleural effusion. Select the best course of action.

Select one:
a. Insertion of a chest tube.
b. Administration of intraarterial vasopressin.
c. Left thoracotomy, full-thickness suture ligation, and drainage of the pleural cavity. !

d. Administration of intravenous vasopressin.

e. Balloon tamponade.

The correct answer is: Left thoracotomy, full-thickness suture ligation, and drainage of the pleural cavity.

A 72-year-old man with severe coronary artery disease presents with painless hematemesis following a prolonged
bout of vomiting. Upper endoscopy reveals a tear just below the gastroesophageal junction, which is actively
bleeding. Select the best course of action.

Select one:
a. Administration of intraarterial vasopressin.
b. Insertion of a chest tube.
c. Administration of intravenous vasopressin.
d. Balloon tamponade. !

e. Left thoracotomy, full-thickness suture ligation, and drainage of the pleural cavity.

The correct answer is: Balloon tamponade.

A diagnosis of bleeding esophageal varices is made in patient. The appropriate initial therapy would be:

Select one:
a. Endoscopic sclerotherapy. !

b. Intravenous vasopressin.
c. Esophageal balloon tamponade.
d. Emergency esophageal transection.
e. Emergency portacaval shunt.

The correct answer is: Endoscopic sclerotherapy.

All of the following problems commonly occur with the use of balloon tamponade for control of variceal bleeding
EXCEPT

Select one:
a. Gastritis. !

b. Pneumonia.
c. Esophageal ulceration or perforation.
d. Rebleeding following removal of the tube.
e. Aspiration of nasopharyngeal secretion.

The correct answer is: Gastritis.

All of the following statements regarding lower GI bleeding are true EXCEPT

Select one:
a. Only 10%-15% of patients stop bleeding spontaneously. !

b. Persistent bleeding is an indication for surgery.


c. The mortality rate is about 10%.
d. If bleeding is profuse, angiography may be useful.
e. Blind total colectomy may be a necessary procedure.

The correct answer is: Only 10%-15% of patients stop bleeding spontaneously.

Dieulafoys lesion of the stomach is characterized by:

Select one:
a. A large mucosal defect with underlying, friable vascular plexus.

b. Frequent rebleeding after endoscopic treatment.


c. Acid-peptic changes of the gastric mucosa.
d. Massive bleeding that requires subtotal gastrectomy.
e. Location in the proximal stomach. !

The correct answer is: Location in the proximal stomach.

Gastric acid production is altered by all of the following hormones or actions EXCEPT:

Select one:
a. Secretin.

b. Vagal stimulation.
c. Glucagon. !

d. Gastrin.
e. Cholecystokinin.

The correct answer is: Glucagon.

Massive upper GI bleeding occurs in an otherwise asymptomatic, normal man following a violent episode of retching
and vomiting without blood. The most likely cause of this man?s bleeding is:

Select one:
a. Mallory-Weiss tear. !

b. Hiatal hernia.
c. Gastritis.
d. Carcinoma of the stomach.
e. Duodenal ulcer.

The correct answer is: Mallory-Weiss tear.

Omeprazole has been added to the H2-antagonists as a therapeutic approach to the management of acute bleeding
gastric and duodenal ulcers. It acts by

Select one:
a. Blocking breakdown of mucosal-damaging metabolites of NSAIDs.
b. Providing a direct cytoprotective effect.

c. Inhibiting gastrin release and parietal cell acid production.


d. Inhibiting parietal cell hydrogen-potassium-ATPase. !

e. Buffering gastric acids.

The correct answer is: Inhibiting parietal cell hydrogen-potassium-ATPase.

The blood supply to the stomach and duodenum arises from all of the following arteries EXCEPT:

Select one:
a. Inferior mesenteric artery.
b. Common hepatic artery.

c. Superior mesenteric artery.


d. Splenic artery.
e. Gastroepiploic artery. !

The correct answer is: Gastroepiploic artery.

The commonest cause of death in patients with alcoholic cirrhosis following portosystemic shunting is:

Select one:
a. Bleeding esophageal varices.
b. Cardiac failure with peripheral edema and ascites.

c. Hepatocellular carcinoma.
d. Hepatic failure with encephalopaty. !

e. Malnutrition.

The correct answer is: Hepatic failure with encephalopaty.

The correct surgical treatment for Mallory-Weiss tear of the esophagus is:

Select one:
a. Laparotomy and resection of the gastroesophageal junction.
b. Antibiotics and observation.
c. Transthoracic antireflux procedure.

d. Laparotomy, gastrotomy, and oversewing the bleeding vessel. !

e. Transthoracic ligation of varices.

The correct answer is: Laparotomy, gastrotomy, and oversewing the bleeding vessel.

The most common cause of massive upper gastrointestinal bleeding is:

Select one:
a. Gastric carcinoma.
b. Erosive gastritis.
c. Duodenal ulcer. !

d. Gastric ulcer.
e. Mallory-Weiss tear.

The correct answer is: Duodenal ulcer.

What is the most reliable method for precisely locating an upper gastrointestinal lesion that is responsible for a
bleeding?

Select one:
a. Arteriography.
b. Upper GI series.
c. Exploratory laparotomy.

d. Upper GI endoscopy. !

e. Radionuclide scanning.

The correct answer is: Upper GI endoscopy.

Which of the following statements regarding stress ulceration is true?

Select one:
a. Increased secretion of gastric acid has been shown to play a causative role.
b. It classically involves the antrum.
c. It is seen following shock or sepsis, but for some unknown reason does not occur following major surgery,
trauma, or burns.
d. It is true ulceration, extending into and through the muscularis mucosa.
e. It frequently involves multiple sites. !

The correct answer is: It frequently involves multiple sites.

Which vessel is most commonly associated with a posterior duodenal ulcer?

Select one:
a. Middle colic artery.
b. Common hepatic artery.
c. Gastroduodenal artery. !

d. Right gastroepiploic artery.


e. Superior mesenteric artery.

The correct answer is: Gastroduodenal artery.

Finish review

◀︎ 08.2 Jump to... 09 ▶︎

Quiz navigation
1 2 3 4 5 6 7 8 9 10 11 12 13 14 15 16 17 18 19 20

Show one page at a time


Finish review

Copyright © 2022 Technomatix Ltd. | All Rights Reserved


( Distance Learning % &

" Dashboard Dashboard / My courses / 6 Surgery with pediatric surgery and oncology
/ 11 Abdominal trauma. Signs of the abdominal and retroperitoneal organs lesions. Instrumental methods of diagnosis.
Diagnostic and medical tactics.
! My Training / Test 11

" Catalog

# Calendar
Started on Saturday, 15 October 2022, 10:01 AM
$ Reports State Finished
Completed on Saturday, 15 October 2022, 10:09 AM
Time taken 8 mins 27 secs
# 6 Surgery with pediatric
surgery and oncology Marks 20.00/20.00
Grade 3.00 out of 3.00 (100%)
$ Participants
18-year-old high school football player is kicked in the left flank. Three hours later he develops hematuria. His vital
% Badges signs are stable. The diagnostic tests performed reveal extravasation of contrast into the renal parenchyma.
Treatment should consist of
& Competencies
Select one:
' Grades a. Nephrostomy.
b. Exploration and wedge resection of the left kidney.
c. Antibiotics and serial monitoring of blood count and vital signs. !

d. Resumption of normal daily activity excluding sports.


e. Exploration and suture of the laceration.

The correct answer is: Antibiotics and serial monitoring of blood count and vital signs.

A 19-year-old gang member is shot in the abdomen with a .38-caliber revolver. The entry wound is in the
epigastrium, to the left of the midline. The bullet is lodged in the psoas muscle on the right. He is hemodynamically
stable, the abdomen is moderately tender. At exploratory laparotomy for the patient described in the previous
question, examination shows clean, punched-out entrance and exit wound sin the transverse colon. Appropriate
manage is:

Select one:
a. Get left-side gemicolectomy.
b. Drainage of the abdominal cavity.
c. Get a right-side gemicolectomy.

d. Get a sygmostomy.
e. Get a transversestomy. !

The correct answer is: Get a transversestomy.

A 19-year-old gang member is shot once with a .38-caliber revolver. The entry wound is in the left mid-clavicular
line, 2 inches below the nipple. The bullet is lodged in the left paraspinal muscles. He is hemodynamically stable, but
he is drunk and combative and physical examination is difficult to perform. What is optimal tactic?

Select one:
a. Ultrasound investigation.
b. Get a laparotomy. !

c. Computer tomography.
d. X-ray chest examination.

e. Diagnostic peritoneal lavage.

The correct answer is: Get a laparotomy.

A 22-year-old man sustains a gunshot wound to the abdomen. At exploration, an apparently solitary distal small-
bowel injury is treated with resection and primary anastomosis. On postoperative day 7, small-bowel fluid drains
through the operative incision. The fascia remains intact. The fistula output is 300 mL/day and there is no evidence
of intraabdominal sepsis. Correct treatment includes

Select one:
a. Total parenteral nutrition. !

b. Broad-spectrum antibiotics.
c. Early reoperation to close the fistula tract.
d. Somatostatin to lower fistula output.

e. Loperamide to inhibit gut motility.

The correct answer is: Total parenteral nutrition.

A 26-year-old woman has been involved in a car wreck. She has fractures in both upper extremities, facial
lacerations, and no other obvious injuries. Chest x-ray is normal. Shortly thereafter she develops hypotension,
tachycardia, and dropping hematocrit. Her CVP is low. What is it?

Select one:
a. Acute pancreatitis.
b. The classic for traumatic diaphragm rupture.
c. Traumatic shock.
d. Obviously blood loss, it has to be in the abdomen. !

e. Acute mesenteric thrombosis.

The correct answer is: Obviously blood loss, it has to be in the abdomen.

A 27-year-old intoxicated man smashes his car against a tree. He is tender over the left lower chest wall. Chest x-ray
shows fractures of the 8th, 9th, and 10th ribs on the left. He has a blood pressure of 85 over 68 and a pulse rate of
128. What is it?

Select one:
a. The classic for traumatic diaphragm rupture.
b. Hematoma of the rectus sheath.
c. This one is a classic ruptured spleen. !

d. The classic for traumatic rupture of the hepar.


e. The classic for traumatic rupture of the aorta.

The correct answer is: This one is a classic ruptured spleen.

A 31-year-old woman smashes her car against a wall. She has multiple injuries including upper and lower extremity
fractures. Her blood pressure is 75 over 55, with a pulse rate of 110, and a CVP of 0. On physical examination, she
has a tender abdomen, with guarding and rebound on all quadrants. What is it?

Select one:
a. injure of the abdomen wall.
b. Acute diverticulitis.
c. Solid organs are smashed. !

d. Hematoma of the rectus sheath.


e. Injure of the kidney.

The correct answer is: Solid organs are smashed.

A 31-year-old woman smashes her car against a wall. She has multiple injuries including upper and lower extremity
fractures. Her blood pressure is 135 over 75, with a pulse rate of 82. On physical examination she has a tender
abdomen, with guarding and rebound on all quadrants. What is it?

Select one:
a. Traumatic shock/
b. Acute peritonitis.
c. Pancreatoduodenal injuries.
d. Rectus-sheath hematoma.
e. Solid organs will be bleed when smashed. !

The correct answer is: Solid organs will be bleed when smashed.

A 36-year-old man sustains a gunshot wound to the left buttock. He is hemodynamically stable. There is no exit
wound, and an xray of the abdomen shows the bullet to be located in the right lower quadrant. Correct management
of a suspected rectal injury would include

Select one:
a. Endoscopy of the bullet track.
b. Barium studies of the colon and rectum.
c. Angiography.
d. Sigmoidoscopy in the emergency room. !

e. Barium studies of the bullet track.

The correct answer is: Sigmoidoscopy in the emergency room.

A 42-year-old man is stabbed in the belly by a jealous lover. The wound is lateral to the umbilicus, on the left, and
omentum can be seen protruding through it. What is optimal tactics?

Select one:
a. Get a laparotomy. !

b. Ultrasound investigation.
c. Ultrasound investigation.
d. Diagnostic peritoneal lavage.
e. Computer tomography.

The correct answer is: Get a laparotomy.

A 53-year-old man is involved in a high-speed automobile collision. He has moderate respiratory distress. Physical
examination shows no breath sounds over the entire left chest. Percussion is unremarkable. Chest x-ray shows
multiple air fluid levels in the left chest. What is it?

Select one:
a. The classic for traumatic rupture of the aorta.

b. The classic for traumatic rupture of the liver.


c. Hematoma of the rectus sheath.
d. Classic for traumatic diaphragmatic rupture. !

e. This one of the classic ruptured spleen.

The correct answer is: Classic for traumatic diaphragmatic rupture.

An 18-year-old high school football player is kicked in the left flank. Three hours later he develops hematuria. His
vital signs are stable. Initial diagnostic tests in the emergency room should include which of the following?

Select one:
a. Retrograde urethrography.

b. Diagnostic peritoneal lavage.


c. Retrograde cystography. !

d. Arteriography.
e. Intravenous pyelogram.

The correct answer is: Retrograde cystography.

An elderly pedestrian collides with a bicycle-riding pizza delivery man and suffers a unilateral fracture of his pelvis
through the obturator foramen. You would manage this injury by:

Select one:
a. Angiographic visualization of the obturator artery with surgical exploration if the artery is injured or
constricted.
b. Direct surgical approach with internal fixation of the ischial ramus.
c. Short-term bed rest with gradual ambulation as pain allows after 3 days. !

d. Hip spica.
e. External pelvic fixation.

The correct answer is: Short-term bed rest with gradual ambulation as pain allows after 3 days.

Blunt trauma to the abdomen most commonly injures which of the following organs?

Select one:
a. Pancreas.
b. Intestine.
c. Liver.

d. Kidney.
e. Spleen. !

The correct answer is: Spleen.

Correct statements regarding blunt trauma to the liver include which of the following?

Select one:
a. Nonanatomic hepatic debridement, with removal of the injured fragments only, is preferable to resection
along anatomic planes.
b. Major hepatic lacerations that are sutured closed will result in intrahepatic hematomas, hemobilia, and bile
fistulas.
c. Intracaval shunting has dramatically improved survival among patients with hepatic vein injuries.
d. The incidence of intraabdominal infections is significantly lower in patients with abdominal drains. !

e. Hepatic artery ligation for control of bleeding is associated with decreased morbidity and mortality.

The correct answer is: The incidence of intraabdominal infections is significantly lower in patients with abdominal
drains.

Following blunt abdominal trauma, a 12-year-old girl develops upper abdominal pain, nausea, and vomiting. An
upper gastrointestinal series reveals a total obstruction of the duodenum with a “coiled spring” appearance in the
second and third portions. Appropriate management is

Select one:
a. TPN to increase the size of the retroperitoneal fat pad.
b. Gastrojejunostomy.
c. Duodenojejunostomy.
d. Nasogastric suction and observation. !

e. Duodenal resection.

The correct answer is: Nasogastric suction and observation.

In the course of a domestic fight, a 38-year-old obese woman is attacked with a 4-inch-long Swiss blade. In addition
to several superficial lacerations, she was stabbed in the abdomen. She is hemodynamically stable, and does not
have any signs of peritoneal irritation. Correct treatment includes

Select one:
a. Direct board-spectrum antibiotics.

b. X-ray abdomen examination.


c. Total parenteral nutrition.
d. Diagnostic peritoneal lavage.
e. Penetrating abdominal wounds have to be surgically explored. !

The correct answer is: Penetrating abdominal wounds have to be surgically explored.

True statements concerning penetrating pancreatic trauma include:

Select one:
a. The major cause of death is exsanguination from associated vascular injuries. !

b. Management of a ductal injury in the head of the pancreas is pancreaticoduodenectomy.


c. Small peripancreatic hematomas need not be explored to search for pancreatic injury.
d. Most injuries do not involve adjacent organs.
e. Management of a ductal injury to the left of the mesenteric vessels is Roux-en-Y pancreaticojejunostomy.

The correct answer is: The major cause of death is exsanguination from associated vascular injuries.

When operating to repair civilian colon injuries:

Select one:
a. Right-sided colonic wounds should not be repaired primarily.

b. Administration of intravenous antibiotics with aerobic and anaerobic coverage has not been shown to
decrease the incidence of wound infections after repair of colonic injuries.
c. The presence of shock on admission or more than two associated intraabdominal injuries is an absolute !
contraindication to primary colonic repair.
d. Distal sigmoidal injuries should not be repaired primarily.
e. A colostomy should be performed for colonic injury in the presence of gross fecal contamination.

The correct answer is: The presence of shock on admission or more than two associated intraabdominal injuries is
an absolute contraindication to primary colonic repair.

Which of the following conditions is most likely to follow a compression-type abdominal injury?

Select one:
a. Superior mesenteric thrombosis.
b. Avulsion of the splenic pedicle.
c. Diaphragmatic hernia. !

d. Mesenteric vascular injury.

e. Renal vascular injury.

The correct answer is: Diaphragmatic hernia.

Finish review

◀︎ 10.1 Jump to... 11 ▶︎

Quiz navigation
1 2 3 4 5 6 7 8 9 10 11 12 13 14 15 16 17 18 19 20

Show one page at a time


Finish review

Copyright © 2022 Technomatix Ltd. | All Rights Reserved


( Distance Learning % &

" Dashboard Dashboard / My courses / 6 Surgery with pediatric surgery and oncology
/ 12 Specific features of the acute abdominal cavity diseases in the pregnant. Peculiarities of examination. Prevention of
premature births. Manifestation of acute surgical diseases of organs of abdominal cavity in elderly patients.
! My Training / Test 12.1

" Catalog

# Calendar
Started on Saturday, 15 October 2022, 10:09 AM
$ Reports State Finished
Completed on Saturday, 15 October 2022, 10:11 AM
Time taken 1 min 31 secs
# 6 Surgery with pediatric
surgery and oncology Marks 10.00/10.00
Grade 3.00 out of 3.00 (100%)
$ Participants
A 30-year-old primigravida complains of headaches, restlessness, sweating, and tachycardia. She is 18 wk pregnant
% Badges and her blood pressure is 200/120 mm Hg. Appropriate workup might include:

& Competencies Select one:


a. Abdominal CT scan
' Grades
b. Exploratory laparotomy
c. Abdominal ultrasonogram !

d. Mesenteric angiography
e. Head CT scan

The correct answer is: Abdominal ultrasonogram

Choose the incorrect statement

Select one:
a. The erythrocyte sedimentation rate is physiologically decreased !

b. Abdominal wall laxity in late pregnancy might mask the classic signs of peritonitis
c. The location of maximal abdominal pain and tenderness from acute appendicitis migrates superiorly and
laterally as the appendix is displaced by the growing gravid uterus
d. A rigid abdomen with rebound tenderness remains a valid indicator of peritonitis during pregnancy

e. An abdominal mass may be missed on physical examination because of the presence of the enlarged gravid
uterus

The correct answer is: The erythrocyte sedimentation rate is physiologically decreased

Contraindications for Diagnostic Peritoneal Lavage

Select one:
a. Unexplained shock or hypotension
b. Altered sensorium (e.g., closed-head injury, drugs)
c. Pregnancy
d. Equivocal pulmonary embolism !

e. General anesthesia for extra-abdominal procedures

The correct answer is: Equivocal pulmonary embolism

Following disorders may cause diffuse abdominal pain may except:

Select one:
a. Peritonitis

b. Porphyria
c. A. Inflammatory bowel disease
d. Hepatic abscess
e. Rib fracture !

The correct answer is: Rib fracture

Following factor contribute to a mild hypercoagulopathy during pregnancy

Select one:
a. Hypernatremia

b. Anemia
c. Estrogen !

d. Leukocytosis
e. Progesterone

The correct answer is: Estrogen

Gynecologic and obstetric causes of left lower quadrant abdominal pain include following except:

Select one:
a. Ovarian cyst rupture

b. Ruptured ectopic pregnancy


c. Pelvic inflammatory disease
d. Tubo-ovarian abscess
e. Cholangitis !

The correct answer is: Cholangitis

Physiologic alterations of laboratory values during pregnancy include the following except:

Select one:
a. Mild dilutional hypoalbuminemia
b. Mild leukocytosis,

c. Mild hypernatremia !

d. Physiologic anemia,
e. Mildly increased alkaline phosphatase level

The correct answer is: Mild hypernatremia

These are risk factors for ectopic pregnancy except:

Select one:
a. maternal age
b. Better PID therapy !

c. prior complicated abdominal surgery, such as ruptured appendicitis


d. Adnexal, or uterine pathology
e. Tubal surgery

The correct answer is: Better PID therapy

What diagnostic imaging is safe for the fetus?

Select one:
a. Plane X-ray
b. ltrasonography !

c. Intravenous pyelography
d. Computerized tomography
e. Magnetic resonance imaging

The correct answer is: ltrasonography

Which of the following statements regarding appendicitis during pregnancy is correct?

Select one:
a. Appendicitis occurs more commonly in pregnant women than in nonpregnant women of comparable age
b. Appendicitis is the most prevalent extrauterine indication for celiotomy during pregnancy !

c. Noncomplicated appendicitis results in a 20% fetal mortality and premature labor rate

d. Suspected appendicitis in a pregnant woman should be managed with a period of observation of due to the
risks of laparotomy to the fetus
e. The severity of appendicitis correlates with increased gestational age of the fetus

The correct answer is: Appendicitis is the most prevalent extrauterine indication for celiotomy during pregnancy

Finish review

◀︎ 11 Jump to... Test 12.2 ▶︎

Quiz navigation
1 2 3 4 5 6 7 8 9 10

Show one page at a time


Finish review

Copyright © 2022 Technomatix Ltd. | All Rights Reserved


( Distance Learning % &

" Dashboard Dashboard / My courses / 6 Surgery with pediatric surgery and oncology
/ 12 Specific features of the acute abdominal cavity diseases in the pregnant. Peculiarities of examination. Prevention of
premature births. Manifestation of acute surgical diseases of organs of abdominal cavity in elderly patients.
! My Training / Test 12.2

" Catalog

# Calendar
Started on Saturday, 15 October 2022, 6:20 PM
$ Reports State Finished
Completed on Saturday, 15 October 2022, 6:22 PM
Time taken 1 min 41 secs
# 6 Surgery with pediatric
surgery and oncology Marks 10.00/10.00
Grade 3.00 out of 3.00 (100%)
$ Participants
A 65-year-old man who is hospitalized with pancreatic carcinoma develops abdominal distention and obstipation.
% Badges The abdominal radiograph revealed distended loops of intestine. Appropriate management would best be achieved
by:
& Competencies
Select one:
' Grades a. Digital disimpaction of a fecal mass in the rectum
b. Urgent colostomy or cecostomy
c. Diagnostic and therapeutic colonoscopy !

d. Detorsion of the volvulus and colopexy or resection


e. Discontinuation of anticholinergic medications and narcotics and correction of metabolic disorders

The correct answer is: Diagnostic and therapeutic colonoscopy

A 65-year-old previously healthy man presents with severe abdominal pain that came on suddenly. He has
abdominal tenderness and guarding in all four quadrants on physical examination. A radiograph is obtained and
demonstrates a radiolucency under the right hemidiaphragm. What diagnosis is unlikely?

Select one:
a. Perforated transverse colon carcinoma
b. Strangulated hernia with necrotic bowel
c. Ruptured echinococcal liver cyst !

d. Perforated gastric ulcer


e. Perforated diverticulum

The correct answer is: Ruptured echinococcal liver cyst

A 72-year-old patient with an intractable type I ulcer along the incisura with a significant amount of scarring along
the entire length of the lesser curvature. Select the appropriate surgical procedure for this patient

Select one:
a. Vagotomy and antrectomy
b. Antrectomy alone !

c. Proximal gastric vagotomy


d. Vagotomy and gastrojejunostomy

e. Vagotomy and pyloroplasty

The correct answer is: Antrectomy alone

An 80-year-old man is admitted to the hospital complaining of nausea, abdominal pain, distention, and diarrhea. A
cautiously performed transanal contrast study reveals an “apple core” configuration in the rectosigmoid.
Appropriate management at this time would include

Select one:
a. Colonoscopic decompression and rectal tube placement
b. Evaluation of an electrocardiogram and obtaining an angiogram to evaluate for colonic mesenteric ischemia
c. Oral administration of metronidazole and checking a Clostridium difficile titer
d. Saline enemas and digital disimpaction of fecal matter from the rectum

e. Colon resection and proximal colostomy !

The correct answer is: Colon resection and proximal colostomy

An 82-year-old nursing home patient presents to the emergency room with vomiting, abdominal pain, and
distention. A radiograph is obtained and demonstrates a grossly dilated loop of intestine overlying the sacrum in the
shape of an upside down U. Select the likely diagnosis.

Select one:
a. Ruptured spleen
b. Sigmoid volvulus !

c. Perforated gastric ulcer


d. Perforated diverticulum

e. Ruptured echinococcal liver cyst

The correct answer is: Sigmoid volvulus

Causes of pain in the right lower quadrant include following except:

Select one:
a. Ruptured ectopic pregnancy
b. Appendicitis
c. Ovarian cyst rupture
d. Sigmoid volvulus !

e. Ruptured Meckel’s diverticulum

The correct answer is: Sigmoid volvulus

Diagnostic abdominal laparoscopy is contraindicated in which of the following patients?

Select one:
a. A stable patient with a stab wound to the lower chest wall
b. An elderly patient in the intensive care unit suspected of having intestinal ischemia
c. A patient with rebound tenderness following a tangential gunshot wound to the abdomen !

d. A young female with pelvic pain and fever


e. A patient with a mass in the head of the pancreas

The correct answer is: A patient with rebound tenderness following a tangential gunshot wound to the abdomen

Obturator hernias typically are found in

Select one:
a. Pregnant
b. Young boys
c. Adolescents
d. Fat men
e. A. Thin elderly women !

The correct answer is: A. Thin elderly women

Patient 78 years old who has never before been hospitalized is admitted with signs and symptoms typical of a small
bowel obstruction. Which of the following clinical findings would give the most help in ascertaining the diagnosis?

Select one:
a. A leukocyte count of 40,000/µL
b. A pH of 7.5, PCO2 of 50 kPa, and paradoxically acid urine
c. A palpable mass in the pelvis !

d. Coffee-grounds aspirate from the stomach


e. Bloody contains aspirate from the stomach

The correct answer is: A palpable mass in the pelvis

Which statement concerning cholangitis is correct?

Select one:
a. The disease occurs primarily in young, immunocompromised patients

b. Surgery is indicated once the diagnosis of cholangitis is made


c. The diagnosis is suggested by the Charcot triad in elderly !

d. Cholecystostomy is the procedure of choice in affected patients


e. The most common infecting organism is Staphylococcus aureus

The correct answer is: The diagnosis is suggested by the Charcot triad in elderly

Finish review

◀︎ Test 12.1 Jump to... 12 ▶︎

Quiz navigation
1 2 3 4 5 6 7 8 9 10

Show one page at a time


Finish review

Copyright © 2022 Technomatix Ltd. | All Rights Reserved


( Distance Learning % &

" Dashboard Dashboard / My courses / 6 Surgery with pediatric surgery and oncology
/ 13 Modern methods of diagnostic and treatment of cardio-vascular and breathing systems. / Test 13

! My Training

" Catalog
Started on Sunday, 2 October 2022, 9:24 AM
# Calendar State Finished
Completed on Sunday, 2 October 2022, 9:26 AM
$ Reports
Time taken 2 mins 22 secs
Marks 20.00/20.00
# 6 Surgery with pediatric Grade 3.00 out of 3.00 (100%)
surgery and oncology
48-years-old woman with ischemic cardiomyopathy have progressive attacks of stenocardia. On coronary
$ Participants angiography: anterior interventricular coronary artery is stenosed approximately 70% for 0, 7 sm. Other arteries are
without hemodynamic changes. It is an indication for:
% Badges
Select one:
& Competencies a. Coronary artery bypass grafting of one artery
b. Coronary artery bypass grafting of 2-3 arteries
' Grades
c. Balloon angioplasty !

d. Medicinal therapy
e. Transplantation of heart

The correct answer is: Balloon angioplasty

A 25-year-old woman arrives in the emergency room following an automobile accident. She has acute dyspnea with
a respiratory rate of 60 breaths/min. Breathing sounds are markedly diminished on the right side. The first step in
managing the patient should be to:

Select one:
a. Decompression of the right pleural space !

b. Draw arterial blood for blood gas determination


c. Perform pericardiocentesis

d. Take a chest x-ray


e. Administer intravenous fluids

The correct answer is: Decompression of the right pleural space

A 55-year-old man has a severe vomiting and retching spell punctuated by a sharp substernal pain. The initial
management plan should be:

Select one:
a. Serial ECGs to rule out myocardial ischemia
b. Flexible esophagogastroscopy to establish diagnosis.
c. Chest film !

d. Nasogastric tube, antibiotics

e. Left chest tube and spit fistula (cervical esophagostomy)

The correct answer is: Chest film

A previously healthy 20-year-old man is admitted to a hospital with acute onset of mild left-sided chest pain.
General condition is normal. The electrocardiographic findings are normal but chest x-ray shows a 20% left
pneumothorax. Patient needs :

Select one:
a. Thoracostomy and intubation
b. Thoracotomy
c. Barium swallow
d. Tube thoracostomy

e. Observation !

The correct answer is: Observation

A sharp left-sided chest pain appeared in a 35-year-old patient at the time of intense physical activity. Objectively:
the patient is covered with cold sweat; breathing is difficult because of the pain. Auscultation: on the right side the
breath sound is vesicular, on the left side - weakened. Tachycardia, heart rate -100. What is your diagnosis?

Select one:
a. Myocardial infarction
b. Heart attack
c. Intercostal Neuralgia
d. Spontaneous pneumothorax !

e. Pneumonia

The correct answer is: Spontaneous pneumothorax

All of the following complications of myocardial infarction are indications for surgical correction EXCEPT

Select one:
a. ventricular aneurysm
b. papillary muscle dysfunction !

c. ruptured ventricle
d. ventricular premature beats
e. ruptured intraventricular septum

The correct answer is: papillary muscle dysfunction

All of the following symptoms and signs are indicative of a tension pneumothorax EXCEPT

Select one:
a. shifting of the trachea towards the pneumothorax !

b. chest pain
c. hypotension
d. shortness of breath
e. absent breath sounds unilaterally

The correct answer is: shifting of the trachea towards the pneumothorax

Several days following esophagectomy a patient complains of dyspnea and chest tightness. A large pleural effusion
is noted on chest radiograph and thoracocentesis yields milky fluid. What occur with patient?

Select one:
a. Failure of sutures
b. Hydrothorax
c. Chylothorax !

d. Pyothorax
e. Pleurisy

The correct answer is: Chylothorax

Several days following esophagectomy a patient complains of dyspnea and chest tightness. A large pleural effusion
is noted on chest radiograph and thoracocentesis yields milky fluid - chyle. Initial management of this patient
consists of which of the following procedure?

Select one:
a. Immediate operation to ligate the thoracic duct
b. Observation and antibiotics
c. Observation and low-fat diet
d. Immediate operation to repair the thoracic duct
e. Tube thoracostomy and low-fat diet !

The correct answer is: Tube thoracostomy and low-fat diet

Tension pneumothorax has the following features, EXCEPT:

Select one:
a. All of the signs

b. Arterial hypotension.
c. The blunt percussion sound on the side of the lesion. !

d. Displacement of the heart on the side opposite pneumothorax.


e. Distension of the neck veins.

The correct answer is: The blunt percussion sound on the side of the lesion.

The best test for establishing the diagnosis and the degree of myocardial dysfunction is

Select one:
a. Serial ECGs !

b. X-ray
c. Echocardiography
d. Creatine phosphokinase (CPK-MB) fractionation
e. Radionuclide angiography

The correct answer is: Serial ECGs

The usual management of cardiac arrest should include all of the following protocols EXCEPT

Select one:
a. immediate resuscitation, as irreversible brain damage will result after 3-4 minutes of diminished perfusion

b. open-chest cardiac massage !

c. administration of cardiotonic agents


d. establishment of an airway and ventilator support
e. defibrillation, if cardiac arrest is due to ventricular fibrillation

The correct answer is: open-chest cardiac massage

Two weeks following the initial management of chylothorax after esophagectomy there is persistent accumulation of
chyle in the pleural space. Appropriate management at this time includes which of the following procedures?

Select one:
a. Subdiaphragmatic ligation of the thoracic duct !

b. Thoracotomy and repair of the thoracic duct

c. Thoracotomy and abrasion of the pleural space


d. Thoracotomy and ligation of the thoracic duct
e. Neck exploration and ligation of the thoracic duct

The correct answer is: Subdiaphragmatic ligation of the thoracic duct

What is the first aid in a patient with tension pneumothorax?

Select one:
a. Thoracostomy
b. Pleural puncture in 6th intercostal space

c. Tube thoracostomy !

d. Pleural puncture in the 7th intercostal space


e. Bronchoscopy

The correct answer is: Tube thoracostomy

What statement about heartburn is not correct?

Select one:
a. Heartburn is caused by acidic fluid from the stomach washing up into the esophagus.
b. Symptoms are often reduce after a large meal !

c. Symptoms are often worse after a large meal, or after using tobacco, alcohol, or caffeine.
d. It is often accompanied by burping, or symptoms of bloating or gas.
e. The pain of heartburn is often a burning discomfort directly beneath the breastbone.

The correct answer is: Symptoms are often reduce after a large meal

What statement about pneumothorax is correct?

Select one:
a. Chest pain occurs at rest, during sleep or very often with minimal exertion.
b. Symptoms are often worse after a large meal, or after using tobacco, alcohol, or caffeine.
c. It is a collection of air or gas in the pleural cavity of the chest between the lung and the chest wall. !

d. It is often accompanied by burping, or symptoms of bloating or gas.


e. Chest pain follows a specific pattern, occurring when someone engages in hard physical activity or
experiences extreme emotion.

The correct answer is: It is a collection of air or gas in the pleural cavity of the chest between the lung and the chest
wall.

What statement about pneumothorax is not true?

Select one:
a. This disease is mainly diagnosed by the appearance of inflammation. !

b. The symptoms of a pneumothorax are determined by the size of the air leak and the speed by which it
occurs; they may include chest pain in most cases and shortness of breath in many.
c. It may occur spontaneously in people without chronic lung conditions ("primary") as well as in those with lung
disease ("secondary").
d. Is a collection of air or gas in the pleural cavity of the chest between the lung and the chest wall.
e. Most pneumothorax occur after physical trauma to the chest, blast injury, or as a complication of medical
treatment.

The correct answer is: This disease is mainly diagnosed by the appearance of inflammation.

What statement about stable angina is correct?

Select one:
a. Сondition that results from blood accumulating in the pleural cavity.
b. Сhest pain occurs at rest, during sleep or very often with minimal exertion.

c. Symptoms often reduce after a large meal.


d. A condition where the lungs are unable to function properly and maintain the normal processes of oxygen
uptake and carbon dioxide removal
e. Chest pain follows a specific pattern, occurring when someone engages in hard physical activity or !
experiences extreme emotion.

The correct answer is: Chest pain follows a specific pattern, occurring when someone engages in hard physical
activity or experiences extreme emotion.

What statement about tension pneumothorax is not true?

Select one:
a. Particular clinical signs may also be useful in the recognition of tension pneumothorax, such as the presence
of raised jugular venous pressure (distended neck veins).
b. Patients require hospital admission
c. In very severe cases, the respiratory rate falls sharply, with shock and coma.
d. Tension pneumothorax is characterized by rapid breathing, cyanosis, falling blood pressure (hypotension) and
confusion.
e. The healthy side of the chest is not hyperexpanded but shows decreased movement, with increased !
movement on the affected side.

The correct answer is: The healthy side of the chest is not hyperexpanded but shows decreased movement, with
increased movement on the affected side.

What statement about unstable angina is correct?

Select one:
a. Chest pain occurs at rest, during sleep or very often with minimal exertion. !

b. Chest pain follows a specific pattern, occurring when someone engages in hard physical activity or
experiences extreme emotion.
c. The pain is often a burning discomfort directly beneath the breastbone.

d. This is a condition that results from blood accumulating in the pleural cavity.
e. A condition where the lungs are unable to function properly and maintain the normal processes of oxygen
uptake and carbon dioxide removal

The correct answer is: Chest pain occurs at rest, during sleep or very often with minimal exertion.

Finish review

◀︎ 12 Jump to... 13 ▶︎

Quiz navigation
1 2 3 4 5 6 7 8 9 10 11 12 13 14 15 16 17 18 19 20

Show one page at a time


Finish review

Copyright © 2022 Technomatix Ltd. | All Rights Reserved


( Distance Learning % &

" Dashboard Dashboard / My courses / 6 Surgery with pediatric surgery and oncology
/ 14 Chest pain syndromes, respiratory and cardiac insufficiency in diagnosis and differential diagnosis of chest diseases.
Diagnostic program in case of cardiac injuries. Triad of symptoms of the cardiac injuries, tactics of surgical treatment.
! My Training / Test 14.1

" Catalog

# Calendar
Started on Saturday, 15 October 2022, 10:12 AM
$ Reports State Finished
Completed on Saturday, 15 October 2022, 10:15 AM
Time taken 3 mins 17 secs
# 6 Surgery with pediatric
surgery and oncology Marks 20.00/20.00
Grade 3.00 out of 3.00 (100%)
$ Participants
48-years-old woman with ischemic cardiomyopathy have progressive attacks of stenocardia. On coronary
% Badges angiography: anterior interventricular coronary artery is stenosed approximately 70% for 0, 7 sm. Other arteries are
without hemodynamic changes. It is an indication for:
& Competencies
Select one:
' Grades a. Transplantation of heart
b. Balloon angioplasty !

c. Medicinal therapy

d. Coronary artery bypass grafting of 2-3 arteries


e. Coronary artery bypass grafting of one artery

The correct answer is: Balloon angioplasty

A 25-year-old woman arrives in the emergency room following an automobile accident. She has acute dyspnea with
a respiratory rate of 60 breaths/min. Breathing sounds are markedly diminished on the right side. The first step in
managing the patient should be to:

Select one:
a. Perform pericardiocentesis
b. Administer intravenous fluids
c. Draw arterial blood for blood gas determination

d. Decompression of the right pleural space !

e. Take a chest x-ray

The correct answer is: Decompression of the right pleural space

A 55-year-old man has a severe vomiting and retching spell punctuated by a sharp substernal pain. The initial
management plan should be:

Select one:
a. Left chest tube and spit fistula (cervical esophagostomy)
b. Chest film !

c. Flexible esophagogastroscopy to establish diagnosis.


d. Serial ECGs to rule out myocardial ischemia

e. Nasogastric tube, antibiotics

The correct answer is: Chest film

A previously healthy 20-year-old man is admitted to a hospital with acute onset of mild left-sided chest pain.
General condition is normal. The electrocardiographic findings are normal but chest x-ray shows a 20% left
pneumothorax. Patient needs :

Select one:
a. Tube thoracostomy
b. Barium swallow
c. Thoracotomy
d. Thoracostomy and intubation

e. Observation !

The correct answer is: Observation

A sharp left-sided chest pain appeared in a 35-year-old patient at the time of intense physical activity. Objectively:
the patient is covered with cold sweat; breathing is difficult because of the pain. Auscultation: on the right side the
breath sound is vesicular, on the left side - weakened. Tachycardia, heart rate -100. What is your diagnosis?

Select one:
a. Myocardial infarction
b. Heart attack
c. Spontaneous pneumothorax !

d. Intercostal Neuralgia

e. Pneumonia

The correct answer is: Spontaneous pneumothorax

All of the following complications of myocardial infarction are indications for surgical correction EXCEPT

Select one:
a. papillary muscle dysfunction !

b. ruptured intraventricular septum


c. ventricular aneurysm
d. ruptured ventricle
e. ventricular premature beats

The correct answer is: papillary muscle dysfunction

All of the following symptoms and signs are indicative of a tension pneumothorax EXCEPT

Select one:
a. shifting of the trachea towards the pneumothorax !

b. chest pain
c. hypotension
d. absent breath sounds unilaterally
e. shortness of breath

The correct answer is: shifting of the trachea towards the pneumothorax

Several days following esophagectomy a patient complains of dyspnea and chest tightness. A large pleural effusion
is noted on chest radiograph and thoracocentesis yields milky fluid. What occur with patient?

Select one:
a. Hydrothorax
b. Pyothorax
c. Chylothorax !

d. Pleurisy
e. Failure of sutures

The correct answer is: Chylothorax

Several days following esophagectomy a patient complains of dyspnea and chest tightness. A large pleural effusion
is noted on chest radiograph and thoracocentesis yields milky fluid - chyle. Initial management of this patient
consists of which of the following procedure?

Select one:
a. Observation and low-fat diet
b. Immediate operation to ligate the thoracic duct
c. Tube thoracostomy and low-fat diet !

d. Observation and antibiotics


e. Immediate operation to repair the thoracic duct

The correct answer is: Tube thoracostomy and low-fat diet

Tension pneumothorax has the following features, EXCEPT:

Select one:
a. The blunt percussion sound on the side of the lesion. !

b. All of the signs


c. Displacement of the heart on the side opposite pneumothorax.
d. Distension of the neck veins.
e. Arterial hypotension.

The correct answer is: The blunt percussion sound on the side of the lesion.

The best test for establishing the diagnosis and the degree of myocardial dysfunction is

Select one:
a. Echocardiography

b. Serial ECGs !

c. Creatine phosphokinase (CPK-MB) fractionation


d. Radionuclide angiography
e. X-ray

The correct answer is: Serial ECGs

The usual management of cardiac arrest should include all of the following protocols EXCEPT

Select one:
a. immediate resuscitation, as irreversible brain damage will result after 3-4 minutes of diminished perfusion

b. establishment of an airway and ventilator support


c. open-chest cardiac massage !

d. defibrillation, if cardiac arrest is due to ventricular fibrillation


e. administration of cardiotonic agents

The correct answer is: open-chest cardiac massage

Two weeks following the initial management of chylothorax after esophagectomy there is persistent accumulation of
chyle in the pleural space. Appropriate management at this time includes which of the following procedures?

Select one:
a. Thoracotomy and ligation of the thoracic duct
b. Neck exploration and ligation of the thoracic duct

c. Thoracotomy and abrasion of the pleural space


d. Thoracotomy and repair of the thoracic duct
e. Subdiaphragmatic ligation of the thoracic duct !

The correct answer is: Subdiaphragmatic ligation of the thoracic duct

What is the first aid in a patient with tension pneumothorax?

Select one:
a. Pleural puncture in 6th intercostal space
b. Tube thoracostomy !

c. Bronchoscopy
d. Thoracostomy
e. Pleural puncture in the 7th intercostal space

The correct answer is: Tube thoracostomy

What statement about heartburn is not correct?

Select one:
a. Symptoms are often worse after a large meal, or after using tobacco, alcohol, or caffeine.
b. The pain of heartburn is often a burning discomfort directly beneath the breastbone.

c. It is often accompanied by burping, or symptoms of bloating or gas.


d. Symptoms are often reduce after a large meal !

e. Heartburn is caused by acidic fluid from the stomach washing up into the esophagus.

The correct answer is: Symptoms are often reduce after a large meal

What statement about pneumothorax is correct?

Select one:
a. It is often accompanied by burping, or symptoms of bloating or gas.
b. Symptoms are often worse after a large meal, or after using tobacco, alcohol, or caffeine.
c. It is a collection of air or gas in the pleural cavity of the chest between the lung and the chest wall. !

d. Chest pain follows a specific pattern, occurring when someone engages in hard physical activity or
experiences extreme emotion.
e. Chest pain occurs at rest, during sleep or very often with minimal exertion.

The correct answer is: It is a collection of air or gas in the pleural cavity of the chest between the lung and the chest
wall.

What statement about pneumothorax is not true?

Select one:
a. Is a collection of air or gas in the pleural cavity of the chest between the lung and the chest wall.
b. Most pneumothorax occur after physical trauma to the chest, blast injury, or as a complication of medical
treatment.
c. This disease is mainly diagnosed by the appearance of inflammation. !

d. The symptoms of a pneumothorax are determined by the size of the air leak and the speed by which it
occurs; they may include chest pain in most cases and shortness of breath in many.
e. It may occur spontaneously in people without chronic lung conditions ("primary") as well as in those with lung
disease ("secondary").

The correct answer is: This disease is mainly diagnosed by the appearance of inflammation.

What statement about stable angina is correct?

Select one:
a. Сhest pain occurs at rest, during sleep or very often with minimal exertion.
b. A condition where the lungs are unable to function properly and maintain the normal processes of oxygen
uptake and carbon dioxide removal
c. Symptoms often reduce after a large meal.

d. Chest pain follows a specific pattern, occurring when someone engages in hard physical activity or !
experiences extreme emotion.
e. Сondition that results from blood accumulating in the pleural cavity.

The correct answer is: Chest pain follows a specific pattern, occurring when someone engages in hard physical
activity or experiences extreme emotion.

What statement about tension pneumothorax is not true?

Select one:
a. Tension pneumothorax is characterized by rapid breathing, cyanosis, falling blood pressure (hypotension) and
confusion.
b. In very severe cases, the respiratory rate falls sharply, with shock and coma.
c. Patients require hospital admission
d. Particular clinical signs may also be useful in the recognition of tension pneumothorax, such as the presence
of raised jugular venous pressure (distended neck veins).
e. The healthy side of the chest is not hyperexpanded but shows decreased movement, with increased !
movement on the affected side.

The correct answer is: The healthy side of the chest is not hyperexpanded but shows decreased movement, with
increased movement on the affected side.

What statement about unstable angina is correct?

Select one:
a. This is a condition that results from blood accumulating in the pleural cavity.
b. A condition where the lungs are unable to function properly and maintain the normal processes of oxygen
uptake and carbon dioxide removal
c. Chest pain follows a specific pattern, occurring when someone engages in hard physical activity or
experiences extreme emotion.
d. The pain is often a burning discomfort directly beneath the breastbone.

e. Chest pain occurs at rest, during sleep or very often with minimal exertion. !

The correct answer is: Chest pain occurs at rest, during sleep or very often with minimal exertion.

Finish review

◀︎ 13 Jump to... Test 14.2 ▶︎

Quiz navigation
1 2 3 4 5 6 7 8 9 10 11 12 13 14 15 16 17 18 19 20

Show one page at a time


Finish review

Copyright © 2022 Technomatix Ltd. | All Rights Reserved


( Distance Learning % &

" Dashboard Dashboard / My courses / 6 Surgery with pediatric surgery and oncology
/ 14 Chest pain syndromes, respiratory and cardiac insufficiency in diagnosis and differential diagnosis of chest diseases.
Diagnostic program in case of cardiac injuries. Triad of symptoms of the cardiac injuries, tactics of surgical treatment.
! My Training / Test 14.2

" Catalog

# Calendar
Started on Saturday, 15 October 2022, 10:15 AM
$ Reports State Finished
Completed on Saturday, 15 October 2022, 10:18 AM
Time taken 3 mins 19 secs
# 6 Surgery with pediatric
surgery and oncology Marks 20.00/20.00
Grade 3.00 out of 3.00 (100%)
$ Participants
A 26-year-old man is brought to the emergency room after being extricated from the driver’s seat of a car involved
% Badges in a head-on collision in which the patient was not wearing his seat belt. His ECG is shown above. The best test for
establishing the diagnosis and the degree of myocardial dysfunction is
& Competencies
Select one:
' Grades a. Radionuclide angiography !

b. Serial ECGs
c. Echocardiography

d. Creatine phosphokinase (CPK-MB) fractionation


e. Coronary angiography

The correct answer is: Radionuclide angiography

A 26-year-old man is brought to the emergency room after being removed from the driver’s seat of a car involved in
a head-on collision in which the patient was not
wearing his seat belt. His ECG is shown below. The ECG is most consistent with

Select one:
a. Chagas disease
b. Myocardial contusion that resulted from the accident !

c. Myocardial ischemia that caused the accident

d. Normal variant
e. Preexisting disease

The correct answer is: Myocardial contusion that resulted from the accident

A 35-year-old man presents with a history of 4 days of severe substernal pain and fever to 38.89°C (102°F). He has
a past medical history of peptic ulcer
disease that resulted in a Billroth II procedure 5 years earlier. On admission, the chest film below is obtained. What is
it?

Select one:
a. Pneumopericardium !

b. The condition may be managed with antibiotics and close observation if the patient remains hemodynamically
stable
c. Pericardial effusion is present
d. The previous Billroth II procedure effectively rules out peptic ulcer as the cause of the condition
e. The condition could have resulted from recurrent peptic ulcer disease

The correct answer is: Pneumopericardium

A 40-year-old woman with generalized myasthenia gravis is being treated with pyridostigmine (Mestinon), a
cholinesterase inhibitor. CT scan shows no evidence of a thymoma. The next step in management of this patient
should be:

Select one:
a. Continued treatment with pyridostigmine
b. High-dose steroid (prednisone) therapy
c. Plasmapheresis followed by thymectomy !

d. Serial CT scans and thymectomy if a thymoma is detected

The correct answer is: Plasmapheresis followed by thymectomy

A 54-year-old woman crashes her car against a telephone pole at high speed. On arrival at the ER she is breathing
well. She has multiple bruises over the chest,
and is exquisitely tender over the sternum at a point where there is a gritty feeling of bone grating on bone, elicited
by palpation. What is it?

Select one:
a. Traumatic rupture of the stomach

b. Sternal fracture with myocardial contusion !

c. Traumatic rupture of the aorta.


d. Traumatic rupture of the lung
e. Traumatic rupture of the gullet

The correct answer is: Sternal fracture with myocardial contusion

Correct statements about thymic tumors (thymomas) include which of the following?

Select one:
a. Their pathology is best determined at surgical exploration

b. They have a female preponderance !

c. Most tumors in adults are malignant


d. They are among the commonest mediastinal tumors

The correct answer is: They have a female preponderance

Cysts are the mesothelial origin and are located in the middle mediastinum, usually at the cardiophrenic angle

Select one:
a. Bronchogenic cyst
b. Teratoma
c. Neuroblastomas

d. Pericardial cyst !

e. Thymoma

The correct answer is: Pericardial cyst

Embriologically, the thymus gland is related to the

Select one:
a. Superior parathyroid glands !

b. Third brachial pouch


c. Fourth brachial pouch

d. Inferior parathyroid glands

The correct answer is: Superior parathyroid glands

Highly malignant tumors, arising from the sympathetic chain in the posterior mediastinum

Select one:
a. Teratoma
b. Pericardial cyst
c. Neuroblastomas !

d. Thymoma

e. Bronchogenic cyst

The correct answer is: Neuroblastomas

Middle mediastinal masses include all of the following EXCEPT

Select one:
a. Bronchogenic cyst
b. Ganglioneuroma !

c. Lymphoma
d. Ascending aortic aneurysm
e. Pericardial cyst

The correct answer is: Ganglioneuroma

Recommended treatment for malignant thymomas includes

Select one:
a. Immunotherapy with monoclonal anti-T-cell antibody !

b. Chemotherapy
c. Radiation therapy
d. Resection of the entire tumor, including areas of invasion

The correct answer is: Immunotherapy with monoclonal anti-T-cell antibody

Regarding myocardial contusion from blunt chest trauma, which of the following statements is correct?

Select one:
a. First-pass radionuclide angiography (RNA) and echocardiography are considered the gold standard for !
diagnosis
b. The majority of patients have abnormalities on the initial ECG post injury

c. Elevated cardiac isoenzyme levels sensitively identify patients at risk for life-threatening arrhythmias
d. All patients diagnosed with myocardial contusion should be monitored in an intensive care unit setting for 72
h
e. RNA and echocardiography are good predictors of subsequent cardiac complications such as arrhythmias
and pump failure

The correct answer is: First-pass radionuclide angiography (RNA) and echocardiography are considered the gold
standard for diagnosis

Select the proper intervention for pericardial tamponade

Select one:
a. Cricothyroidotomy
b. Tube thoracostomy
c. Occlusive dressing
d. Endotracheal intubation

e. Subxiphoid window !

The correct answer is: Subxiphoid window

The patient of 42 was taken in 3 hours after a trauma with the expressed hypodermic emphysema of overhead half
of a body, shortness of breath, tachycardia,
and pulse-120. Roentgenologic did not discover pneumotorax, mediastinum is considerably extended in both sides.
What is the first aid?

Select one:
a. Draining of superior mediastinum
b. Draining of anterior mediastinum !

c. Draining of inferior mediastinum


d. Draining of posterior mediastinum
e. Thoracotomy

The correct answer is: Draining of anterior mediastinum

The superior vena cava syndrome is most frequently seen in association with Histoplasmosis (sclerosing
mediastinitis)

Select one:
a. Bronchiectasis
b. Constrictive pericarditis
c. Thoracic aortic aneurysm
d. Bronchogenic carcinoma !

e. Substernal thyroid

The correct answer is: Bronchogenic carcinoma

Tumors contain elements of all three germ cell layers and frequently calcify

Select one:
a. Thymoma
b. Bronchogenic cyst
c. Neuroblastomas
d. Teratoma !

e. Pericardial cyst

The correct answer is: Teratoma

Tumors occur in the anterior mediastinum and are associated with the clinical syndrome of myasthenia gravis

Select one:
a. Pericardial cyst
b. Neuroblastomas
c. Thymoma !

d. Bronchogenic cyst
e. Teratoma

The correct answer is: Thymoma

Tumors primary to the thymus gland may be associated with which of the following disorders?

Select one:
a. Stricture of the esophagus

b. Scleroderma
c. Lupus erythematosus
d. Myasthenia gravis !

e. Pericarditis

The correct answer is: Myasthenia gravis

Which of the following situations would be an indication for performance of a thoracotomy in the emergency room?

Select one:
a. Penetrating thoracic trauma and no signs of life in the field

b. Rapidly deteriorating patient with cardiac tamponade from penetrating thoracic trauma !

c. Blunt trauma to multiple organ systems with obtainable vital signs in the field but none on arrival in the
emergency room
d. Penetrating abdominal trauma and no signs of life in the field
e. Massive hemothorax following blunt trauma to the chest

The correct answer is: Rapidly deteriorating patient with cardiac tamponade from penetrating thoracic trauma

Which part of mediastinum doesn’t exist

Select one:
a. Superior
b. Middle !

c. Inferior
d. Anterior
e. Posterior
f. Front

The correct answer is: Middle

Finish review

◀︎ Test 14.1 Jump to... 14.1 ▶︎

Quiz navigation
1 2 3 4 5 6 7 8 9 10 11 12 13 14 15 16 17 18 19 20

Show one page at a time


Finish review

Copyright © 2022 Technomatix Ltd. | All Rights Reserved


) Distance Learning % &

# Dashboard Dashboard / My courses / 6 Surgery with pediatric surgery and oncology


/ 15 Traumatic damages of the thorax. Clinical picture and diagnosis of pneumothorax, hemothorax and chylothorax. Tactics
of surgical treatment. Surgical diseases of lungs and pleura. Principles of diagnosis and treatment.
! My Training / Test 15.1

" Catalog

# Calendar
Started on Sunday, 2 October 2022, 9:27 AM
$ Reports State Finished
Completed on Sunday, 2 October 2022, 9:29 AM
Time taken 2 mins 33 secs
$ 6 Surgery with pediatric
surgery and oncology Marks 19.00/20.00
Grade 2.85 out of 3.00 (95%)
% Participants
A 27-year-old patient was diagnosed with a traumatic rib fracture. After 24 hours, the accumulation of exudate in
& Badges the pleural cavity was clinically and radiologically suspected, but no exudate was obtained during pleural puncture.
Probable cause:
' Competencies
Select one:
( Grades a. Incorrect puncture site selected.
b. No damage.
c. Collapsing hemothorax. !

d. The puncture needle is too deep.


e. Pulmonary clogging.

The correct answer is: Collapsing hemothorax.

A 32-year-old patient with a closed chest injury had subcutaneous emphysema in the area of the abdomen, which
quickly spread to the neck, face, upper body. Determine the mechanism of such development of subcutaneous
emphysema:

Select one:
a. Fracture of ribs.
b. Respiratory damage with development of mediastinal emphysema. !

c. Closed pneumothorax.

d. Aperture rupture.
e. Heartbeat.

The correct answer is: Respiratory damage with development of mediastinal emphysema.

A patient 33 years old was taken to the surgical ward within an hour after a car injury (thorax). The patient's
condition is severe. Skin and mucous membranes pale, blood pressure - 80/40 mm Hg. in., pulse 110 beats. per
minute there is a swelling of the neck veins. Ultrasound revealed hemopericardium. What should be the surgical
tactics?

Select one:
a. Pericardial puncture
b. Thoracotomy, or sternotomy !

c. Drainage of the pleural cavity in the fifth intercostal space on the axillary lines on the left
d. Thoracoscopy

e. Pleural puncture in the second intercostal space along the midclavicular line to the left

The correct answer is: Thoracotomy, or sternotomy

A reliable sign of aperture rupture is:

Select one:
a. Chest pain with irradiation in the upper arm
b. Hemorrhage pain
c. Relaxation of breathing on the side of trauma
d. The penetration of abdominal organs into the thoracic cavity revealed by radiological examination !

e. General serious condition of the patient

The correct answer is: The penetration of abdominal organs into the thoracic cavity revealed by radiological
examination

Choose a method of surgical treatment of a penetrating chest wound:

Select one:
a. Sewing all layers of the wound. "

b. Tamponade with sterile tissue.


c. Early delayed suture of the wound.
d. Sewing all layers of the wound.

e. Open wound healing.


f.
g. Stitching the muscles and leaving the skin unprotected.

The correct answer is: Sewing all layers of the wound.

How much hemothorax is subject to mandatory evacuation by draining the pleural cavity
_____

Select one:
a. Up to 100 ml
b. Up to 300 ml
c. Up to 200 ml.

d. More than 300 ml !

e. Up to 50 ml

The correct answer is: More than 300 ml

Patient P., 48, turned to another surgeon complaining of pain in the right half of the chest, shortness of breath,
cough, dizziness. From the anamnesis: an hour ago, he fell off a motorcycle and struck the right side of his chest at
the side of the sidewalk. Objectively: skin and visible mucous membranes are pale, consciousness is blurred, blood
pressure is 90/60 mm Hg. art., pulse - 100 for 1 min, in the chest right breathing weakened, percussion - dullness in
the lower sections. Note the pathological process that has developed in the patient:
_____

Select one:
a. Fracture of the ribs on the right.
b. Right-sided pyopneumothorax.
c. Right-sided tension pneumothorax.

d. Right-sided hemothorax. !

e. Right-sided closed pneumothorax.

The correct answer is: Right-sided hemothorax.

Patient, 25 years old, hospitalized after chest injury. After clinical and radiological examination, the diagnosis was
diagnosed: "Tumor pneumonia left." Specify the first aid method:

Select one:
a. The use of analgesics.
b. Intubation.
c. Intravenous infusions.

d. Oxygen therapy.
e. Drainage of the pleural cavity. !

The correct answer is: Drainage of the pleural cavity.

Physical signs of spontaneous pneumothorax:

Select one:
a. No respiratory noise on one side.
b. Dry wheezing over the affected lung.
c. Cyanosis. !

d. Wet rales over the affected lung.

e. The appearance of subcutaneous emphysema.

The correct answer is: Cyanosis.

Specify an unusual sign of intense pneumothorax:


_____

Select one:
a. Anuria, oliguria.
b. Expansion of the cervical veins.
c.
d. Chest pain.

e. Tracheal deviation toward pneumothorax. !

f. Absence of respiratory movements on the side of the lesion.

The correct answer is: Tracheal deviation toward pneumothorax.

Specify the type of pneumothorax that is especially life-threatening:

Select one:
a. Open.
b. Closed.
c. Strained. !

d.
e. Everyone is equally dangerous.
f. It cannot be life-threatening.

The correct answer is: Strained.

The most advantageous place for drainage of the pleural cavity to eliminate hemothorax
_____

Select one:
a. 2nd intercostal space along the middle clavicular line
b. 7th intercostal space along the anterior axillary line
c. 8th intercostal space along the midline.

d. 4th intercostal space along the middle axillary line


e. 6th intercostal space along the middle axillary line. !

The correct answer is: 6th intercostal space along the middle axillary line.

The most appropriate method of pleural drainage (suction) in the case of spontaneous pneumothorax:

Select one:
a. Passive airless suction.
b. Active vacuum suction.
c. Active vacuum suction in the fifth intercostal space along the midclavicular line.

d.
e. Passive suction valve (Byulau). !

f. Thoracotomy, revision of the pleural cavity, drainage.

The correct answer is: Passive suction valve (Byulau).

The most common cause of spontaneous pneumothorax is:

Select one:
a. Chronic inflammatory processes in the lung.
b. Bullous emphysema. !

c. Emphysema of the lungs.


d. Bronchitis, bronchial asthma.
e. Separation of pleural leaves.

The correct answer is: Bullous emphysema.

The most favorable place for drainage of the pleural cavity to eliminate pneumothorax

Select one:
a. 7th intercostal space along the anterior axillary line
b. 4th intercostal space along the middle axillary line
c. 2nd intercostal space in the middle clavicular line !

d. 8th intercostal space along the posterior axillary line.


e. 3rd intercostal space along the middle clavicular line.

The correct answer is: 2nd intercostal space in the middle clavicular line

The tactics of the surgeon in case of uninformative radiological examination in the chest injury:

Select one:
a. Perform thoracoscopy.
b. Be guided by percussion data.
c. Assess the severity of blood loss on clinical grounds.

d. Be guided by auscultation data.


e. Perform diagnostic pleural puncture. !

The correct answer is: Perform diagnostic pleural puncture.

What are the indications for explorative (exploratory) thoracotomy in patients with relatively stable hemodynamics in
the case of bleeding from pleural drainage:

Select one:
a. 1500 ml were dispensed at once !

b. Removal of more than 100 ml per hour for more than 4 hours
c.
d. 300 ml were dispensed at once.

e. 1000 ml were dispensed at once


f. 500 ml were dispensed at once.

The correct answer is: 1500 ml were dispensed at once

What is meant by the term "floating" rib fracture?

Select one:
a. Multiple rib fractures
b. Severe form of pneumothorax
c. Chest trauma combined with acute respiratory failure
d. Fracture of ribs with lung injury.

e. Fractures of three or more adjacent ribs in two or more places, which lead to loss of their connection with !
the costal framework

The correct answer is: Fractures of three or more adjacent ribs in two or more places, which lead to loss of their
connection with the costal framework

What is the most effective way to eliminate the pain of a rib fracture

Select one:
a. Oral administration of painkillers
b. Intercostal blockage
c.
d. Epidural thoracic anesthesia !

e. Application of lidocaine patch

f. Intravenous use of painkillers

The correct answer is: Epidural thoracic anesthesia

What tube diameter should be used for drainage of the pleural cavity in the case of pneumothorax / haemothorax

Select one:
a. 42 Fr

b. 14 Fr
c. 16 Fr
d. 36 Fr
e. 28 Fr !

The correct answer is: 28 Fr

Finish review

◀︎ Cardialgia Jump to... Test 15.2 ▶︎

Quiz navigation
1 2 3 4 5 6 7 8 9 10 11 12 13 14 15 16 17 18 19 20

Show one page at a time


Finish review

Copyright © 2022 Technomatix Ltd. | All Rights Reserved


( Distance Learning % &

" Dashboard Dashboard / My courses / 6 Surgery with pediatric surgery and oncology
/ 15 Traumatic damages of the thorax. Clinical picture and diagnosis of pneumothorax, hemothorax and chylothorax. Tactics
of surgical treatment. Surgical diseases of lungs and pleura. Principles of diagnosis and treatment.
! My Training / Test 15.2

" Catalog

# Calendar
Started on Sunday, 2 October 2022, 9:30 AM
$ Reports State Finished
Completed on Sunday, 2 October 2022, 9:32 AM
Time taken 2 mins 36 secs
# 6 Surgery with pediatric
surgery and oncology Marks 20.00/20.00
Grade 3.00 out of 3.00 (100%)
$ Participants
A 36-year-old patient, admitted to the hospital with complaints of cough with decaying phlegm (up to 150 ml per
% Badges day), hemoptysis, periodically increase of temperature to 37,8 С, loss of appetite, general weakness. He is ill for 10
years, exacerbation of the disease in spring and in autumn. Objectively: pallor, mild аcrocyanosis, bulging of nail
& Competencies phalanxes of fingers. Auscultation: moist rales under the left scapular. X-ray shows expansion of the lung root,
cellular-like structure of the lung on the left. What is the most reliable method for clarification of the diagnosis?
' Grades
Select one:
a. Plain x-ray.
b. Bronchoscopy.
c. Bronchography. !

d. Tomography.
e. Thorascopy.

The correct answer is: Bronchography.

A 40-year-old male, slept in state of acute alcoholic intoxication for 4-5 hours outdoor. Two days later his
temperature rose to hectic fever and chest pain appeared. In 2 weeks suddenly about 200 ml of pus with unpleasant
smell excretes with cough. What is your diagnosis?

Select one:
a. Bronchoectatic disease.
b. Exacerbation of chronic bronchitis.
c. Acute abscess of the lung. !

d. Cancer of the lung with development of pneumonitis.


e. Pleurisy.

The correct answer is: Acute abscess of the lung.

A 42-year-old patient has been transferred to the surgical department from a therapeutic department, where during
two months from the onset of the disease he was treated for acute abscess of the apex of the right lung. The
improvement of the clinical course was insignificant. There is cough with 80-100 ml of phlegm daily which has
unpleasant smell, temperature is about 37, 6 С. What is your diagnosis?

Select one:
a. Tubercular cavity.
b. Acute abscess of the right lung.
c. A hollow form of lung cancer.
d. Festering polycystosis.

e. Chronic pulmonary abscess. !

The correct answer is: Chronic pulmonary abscess.

A 45-year-old patient who has been treated for an acute abscess of the left lung suddenly felt severe pain in the left
hemithorax and stuffiness when coughing. Physical examination shows that tachycardia is present. Thoracic X-ray
revealed the collapse of left-lung and some air in the left pleura cavity with the horizontal level of liquid. What is the
origin of this complication?

Select one:
a. Acute сardio-pulmonary insufficiency.
b. Passing of inflammation in to the visceral pleura.
c. Atelectasis of the left lung.
d. Rupture of the bulla of the left lung.

e. Rupture of the abscess in pleural cavity. !

The correct answer is: Rupture of the abscess in pleural cavity.

A 78-year-old patient, admitted to the hospital with the complaints on the pain in the left half of the thorax, cough of
80 ml of mucopurulent phlegm daily, increase of temperature to 37, 2 С. Chest x-ray shows a cavity with a polycyclic
internal contour and star-like external one and insignificant infiltration around in the lower part of the lung. What is
your diagnosis?

Select one:
a. Cavernous form of lung cancer. !

b. Chronic abscess of the lung.


c. Festering cyst of the lung.
d. Cavernous tuberculosis.

e. Limited empyema of pleura.

The correct answer is: Cavernous form of lung cancer.

A correct statement concerning bronchial carcinoid tumors is:

Select one:
a. They rarely produce the carcinoid syndrome. !

b. Five-year survival is less than 50%.


c. They frequently metastasize.
d. They most commonly arise in peripheral terminal bronchioles.
e. They are radiosensitive.

The correct answer is: They rarely produce the carcinoid syndrome.

A man of 40 has slept outside for 4-5 hours after being drunk. In 2 hours his body temperature increased, pains in
chest appeared. Temperature gradually increased to 39. In 2 weeks during cough suddenly 200ml foul smelling pus
appeared. What’s the most possible diagnosis?

Select one:
a. Pleurisy.
b. Bronchiectatic disease.
c. Cancer of lung with a development of pneumonitis.
d. Exacerbation of chronic bronchitis.
e. Acute abscess of lungs. !

The correct answer is: Acute abscess of lungs.

At the time of operation on the patient in the earlier question, a firm, rubbery lesion in the periphery of the lung is
discovered. It is sectioned in the operating room to reveal tissue that looks like cartilage and smooth muscle. The
most likely diagnosis is:

Select one:
a. Fibroma.
b. Osteochondroma.
c. Chondroma.
d. Aspergilloma.
e. Hamartoma. !

The correct answer is: Hamartoma.

In 39-year-old patient, became ill after supercooling. His temperature rose to 40 С. Pain appeared in a left side of
the thorax, vomiting with foul-smelling phlegm. Patient abuses tobacco and alcohol. Physical examination reveals:
severe general state, temperature is 39 С, expressed signs of festering intoxication and respiratory insufficiency.
Roentgenologically: on a background massive darkening of the apex and middle lobe of left lung there are the
cavities of different sizes with the horizontal levels of liquid. What is your diagnosis?

Select one:
a. Empyema of pleura.
b. Tubercular cavities.
c. Gangrene of the lung. !

d. Abscess of the lung.


e. Hollow form of the cancer.

The correct answer is: Gangrene of the lung.

In a 62-year-old patient, X-ray of the thorax reveals polycyclic “shade” of the right lung's root, deforming upper part
of the bronchial tube, and atelectasis of apex. Tuberculosis in anamnesis. What research methods may be used for
differentiation between central cancer of the lung and tuberculosis?

Select one:
a. Blood count.

b. Analysis of phlegm on TBC.


c. Bronchoscopy, morphological examination. !

d. Computed tomography.
e. Oncomarkers.

The correct answer is: Bronchoscopy, morphological examination.

Indications for resection in patients with pulmonary tuberculosis include all of the following EXCEPT

Select one:
a. Destroyed lung or lobe.

b. Suspected carcinoma.
c. Residual cavity with positive sputum.
d. Hemoptysis.
e. Residual cavity with negative sputum. !

The correct answer is: Residual cavity with negative sputum.

Management of lung abscess refractory to prolonged antibiotic treatment include all of the following
EXCEPTManagement of lung abscess refractory to prolonged antibiotic treatment include all of the following
EXCEPT~

Select one:
a. Open drainage.

b. Bronchoscopic endobronchial drainage.


c. Tube drainage.
d. Intracavitary antibiotic instillation. !

e. Lobectomy.

The correct answer is: Intracavitary antibiotic instillation.

Specific lung and pleural complication of destructive pneumonia is:

Select one:
a. Lobar emphysema.
b. Bullas.

c. Abscess.
d. Pyothorax. !

e. Cyst of the lung.

The correct answer is: Pyothorax.

Superior pulmonary sulcus carcinomas (Pancoast tumors) are bronchogenic carcinomas that typically produce
which of the following clinical features?

Select one:
a. Nonproductive cough.
b. Horner syndrome. !

c. Atelectasis of the involved apical segment.


d. Hemoptysis.
e. Pain in the T4 and T5 dermatomes.

The correct answer is: Horner syndrome.

The chief diagnostic method for differentiation of pleural effusion is:

Select one:
a. Thoracentesis. !

b. Ultrasound.

c. X-ray.
d. Bronchoscopy.
e. CT.

The correct answer is: Thoracentesis.

The condition shown in the x-rays below is compatible with which of the following manifestations?

Select one:
a. Normal pressure in the body of the esophagus.
b. Higher-than-normal incidence of esophageal carcinoma. !

c. Failure of the upper esophageal sphincter to relax in response to swallowing.

d. Normal esophageal motility.


e. Difficulty swallowing solids but not liquids.

The correct answer is: Higher-than-normal incidence of esophageal carcinoma.

The patient enters to the emergency department with a high temperature (39? С), pain in chest, bad smell from the
mouth, and productive cough with sputum (divides into three layers). What disease can be firstly thought about?

Select one:
a. Pleurisy.
b. Cancer of lung with development of pneumonitis.
c. Acute abscess of lung. !

d. Exacerbation of chronic bronchitis.


e. Bronchiectatic disease.

The correct answer is: Acute abscess of lung.

The rupture of abscess of lung and presence of pus and air in the pleural cavity manifests with all of the following
EXCEPT:

Select one:
a. The compression of lung and displacement of mediastinum to the healthy side. !

b. Tachycardia.
c. Acute pain in the side.

d. Shallow breath.
e. Strong cough.

The correct answer is: The compression of lung and displacement of mediastinum to the healthy side.

True statements about the lesion visualized on the film include which of the following?

Select one:
a. There is a 90% chance that this mass is malignant.
b. It is more apt to be metastatic breast carcinoma than primary lung carcinoma.
c. If the mass is malignant, the possibility for cure with excision is remote.
d. The mass is most likely benign.

e. Since the diagnosis can only be established with certainty by resection, the mass should be excised. !

The correct answer is: Since the diagnosis can only be established with certainty by resection, the mass should be
excised.

What is the most effective way of treatment of patients with acute pleural empyema?

Select one:
a. Draining of pleural cavity with active permanent aspiration. !

b. Draining of pleural cavity by Bulough's system.


c. The injection of antibiotics with proteolytic ferments inside the trachea.
d. Tapping of pleural cavity with removing of pus.

e. Thoracotomy with pleurectomy.

The correct answer is: Draining of pleural cavity with active permanent aspiration.

Finish review

◀︎ Test 15.1 Jump to... 15.1 ▶︎

Quiz navigation
1 2 3 4 5 6 7 8 9 10 11 12 13 14 15 16 17 18 19 20

Show one page at a time


Finish review

Copyright © 2022 Technomatix Ltd. | All Rights Reserved


( Distance Learning % &

" Dashboard Dashboard / My courses / 6 Surgery with pediatric surgery and oncology
/ 16 Syndrome of limb ischemia. Diseases that lead to the syndrome of limb ischemia. Conservative and surgical treatment
of limb arterial occlusion.
! My Training / Test 16

" Catalog

# Calendar
Started on Saturday, 15 October 2022, 6:22 PM
$ Reports State Finished
Completed on Saturday, 15 October 2022, 6:25 PM
Time taken 3 mins 7 secs
# 6 Surgery with pediatric
surgery and oncology Marks 19.00/19.00
Grade 3.00 out of 3.00 (100%)
$ Participants
77-year-old man complaining of pain in the groin, history of femoral artery catheterization after a car accident. On
% Badges examination, there is a large pulsatile formation over the common femoral artery. Skin over mass erythematous.
Duplex ultrasound shows the area of hypoechogenicity surrounding the artery containing active blood flow, which
& Competencies communicates with the underlying artery. Which of the following is the most likely diagnosis?

' Grades Select one:


a. abdominal care
b. pseudoaneurysm !

c. hemangioma

d. hematoma
e. general aneurysm of the common femoral artery

The correct answer is: pseudoaneurysm

A 24-year-old man complains of progressive intermittent limp of the left leg. When examining the PKA, the artery of
the foot, LPG - the pulse is normal; but it disappears with the back flexion of the leg What is the most likely
diagnosis?

Select one:
a. obliterating thromboangitis
b. embolic occlusion
c. cystic degeneration of PKA

d. PKA capture syndrome !

e. obliterative atherosclerosis

The correct answer is: PKA capture syndrome

A 25-year-old woman complains of headache, dizziness and chest pain. She reports that she was once told she had
high blood pressure during pre-exercise. She reports that her mother also had similar symptoms and died of a stroke
at the age of 34. Its temperature is 37.2 ° C, blood pressure 175/90 mm Hg, pulse 65 beats / min, and breathing 18
breaths / min. Examination of the cardiovascular system - without features. Noises are heard on either side of the
navel. Which of the following is the most likely diagnosis?

Select one:
a. essential hypertension
b. fibrovascular dysplasia !

c. atherosclerosis
d. connective tissue disease

e. Kawasaki disease

The correct answer is: fibrovascular dysplasia

A 40-year-old chronic smoker turned with an ulcer of the tip of the second, third and fourth toes of the right foot.
Has a history of recurrent migratory superficial phlebitis of the foot, which occurred several years ago. A medical
examination revealed the absence of pulse on both sides of the arteries of the foot and the LPG. What will be the
single most important step in treatment?

Select one:
a. immediate surgery
b. multiple finger amputations
c. angiography followed by bypass;
d. smoking cessation !

e. long-term anticoagulation therapy

The correct answer is: smoking cessation

A 56-year-old man has a history of leg pain at rest. The patient also has a history of severe CHD. It cannot pass two
steps without breathlessness. The examination revealed an occlusive lesion of the aorta and the iliac arteries. He
needs surgery. Which of the following is acceptable?

Select one:
a. aortic-biliary bypass surgery
b. aortic-iliac angioplasty and stenting
c. axillary-bifemoral bypass !

d. aorto-bifemoral bypass

e. axillary club

The correct answer is: axillary-bifemoral bypass

A 57-year-old man who works as a courier complains of pain in the lower leg while walking, and is only able to walk
for 500 m and then stop. He is concerned that he may lose his job. The patient has diabetes and is taking a tablet of
hypoglycemic medicine, a long-acting β-blocker, and a statin. He smokes a pack of cigarettes every day. On
examination, it has a normal impulse on the right leg. The left pulse is not determined. Which of the following is the
most likely site of arterial occlusion?

Select one:
a. its infrarenal aorta
b. his left superficial femoral artery
c. its anonymous artery
d. its descending thoracic aorta
e. his left iliac artery !

The correct answer is: his left iliac artery

A 57-year-old man with acute onset of right leg pain. He states that he was in his normal state of well-being until 4
hours ago when he felt a sudden onset of pain in his right leg. Then he felt the numbness of his foot. Review: non-
rhythmic heart rate 120 beats / min, blood pressure 130/78 mm Hg. Art. and respiratory rate 24 breaths / minute. On
the cardio monitor irregular rhythm with no p-waves. The right lower extremity is cool to the touch and has a bluish
color below the middle of the thigh. The patient does not feel the toes and cannot move them. His aortic pulse is
normal, there is a ripple on the hip, normal bilateral, on the popliteal and foot - left - unchanged, right - absent.
Which of the following is the most appropriate treatment option?

Select one:
a. systemic heparinization, femoral artery embolectomy !

b. catheter-directed thrombolytic therapy


c. ystemic heparinization, angiography, placement of the correct superficial system of the stent of the femoral
artery
d. systemic heparinization
e. right thigh-hamstring bypass

The correct answer is: systemic heparinization, femoral artery embolectomy

A 60-year-old man complains at a surgical clinic of left knee pain and discomfort that has worsened after standing
for an extended period of time. Physical examination in the subclavian area revealed palpating pulsating mass.
Which of the following is the most common acute complication of this patient problem?

Select one:
a. thromboembolism !

b. distal ischemia is secondary to compartment syndrome


c. formation of hematoma
d. formation of arteriovenous fistula
e. the gap

The correct answer is: thromboembolism

A 60-year-old man with a history of atrial fibrillation found that the lower extremity on the right was cyanotic, cold.
Most likely, embolism comes from which of the following departments?

Select one:
a. lungs

b. abdominal aortic aneurysm


c. paradoxical embolism
d. аtherosclerotic plaque
e. the heart !

The correct answer is: the heart

A 63-year-old man complains of pain and drowsiness of 4 left toes for 2 days. Pulse on a. Dorsalis pedis and LPG
are palpated on both sides. There is no history of heart or vascular disease. What is the most likely diagnosis?

Select one:
a. Raynaud's syndrome
b. cardiac embolism

c. atherosclerosis of the arteries of the fingers


d. atheroembolism !

e. lupus vasculitis

The correct answer is: atheroembolism

A 68-year-old woman complains to a significant surgical clinic pain in her right leg after walking 200 meters. She
reports that the pain is completely resting and can occur again when walking the same distance. She has been
smoking a pack of cigarettes a day for the past 40 years. Her other illnesses include diabetes, obesity, and a recent
urinary tract infection. Her blood pressure today is 130/80 mm Hg. The pulse Doppler on both extremities revealed
KPIs of 0.6 and 0.5 on the left and right ends, respectively. What is the best next step in treatment?

Select one:
a. anticoagulation with warfarin
b. thigh-hamstring bypass

c. observation
d. smoking cessation and exercise !

e. arteriogram of the right leg

The correct answer is: smoking cessation and exercise

A 70-year-old executive complains of intermittent lameness of both legs arising from the passage of the 3rd quarter.
What is the chance for him to develop a gangrene that threatens the limb?

Select one:
a. less than 10% !

b. more than 75%

c. 45%
d. 60%
e. 20%

The correct answer is: less than 10%

A 70-year-old man complains of pain on his right side, irritating to the groin. He has a long history of smoking and
drinking and has reported the passing of kidney stone approximately 20 years before these events. His medical
history is also burdened with diabetes, high cholesterol and obesity. CT shows a 7 mm stone of the right ureter. In
addition, the following image was obtained. Which of the following is a factor? What is the biggest risk factor for
developing this patient's additional findings in the study?

Select one:
a. history of urolithiasis
b. metabolic syndrome
c. smoking history !

d. age
e. male age

The correct answer is: smoking history

A 70-year-old man with a long history of diabetes turned with gangrene to the right on the second toe. What is true
of his claims about his diabetic foot?

Select one:
a. right femoral artery, most likely occluded or stenosed.
b. trophic ulcers are sharply demarcated. !

c. pulse on A. Dorsalis pedis and HPA are always absent.

d. arterial reconstruction is a must


e. the gangrene of the foot always requires immediate amputation below the knee.

The correct answer is: trophic ulcers are sharply demarcated.

A 72-year-old retired banker complains of intermittent limp on the left while playing golf. The angiogram shows
occlusion of the superficial femoral artery and reproduction of the popliteal artery below the knee. What is the
treatment of choice?

Select one:
a. tumor shunt with reversed large subcutaneous vein
b. endarterectomy from the superficial femoral artery
c. in situ femoralplane shunt

d. femomopolite shunt with polytetrofluoroethylene (PTFE) prosthesis


e. energy load program !

The correct answer is: energy load program

A middle-aged person should have an amputation of the left leg due to secondary gangrene STA. Which of the
following statements is true after an amputation below the knee?

Select one:
a. the prognosis of the cult may be determined by transcutaneous oxygen monitoring. !

b. large and small tibia of equal length at this level.


c. it is less functionally effective than knee-level amputation.
d. a poor prognosis is inevitable if the Doppler does not detect a pulse at this level.

e. transaction level 5 cm above medial bone.

The correct answer is: the prognosis of the cult may be determined by transcutaneous oxygen monitoring.

An 82-year-old woman with a history of severe dementia and left brain vessels. GPMK, right hemiparesis. Gangrene
of the toe and erythema of the foot and leg were detected. This woman is severely debilitated by her dementia and
stroke bedridden in a health care setting. On examination, the pulse is weakened on the thigh on both sides, on the
hamstring on the right and there is no hamstring pulse on the left, and the pulse on the foot on both sides is not
palpated. Her left first and second fingers are gangrenosally altered, spilled inflammation of the p / w cellulose of the
left foot and distal third of the lower leg. PCI on the left is 0.25. What is the best treatment option for this patient?

Select one:
a. AH to identify the affected area then perform open bypass surgery, followed by finger amputation and
antibiotic treatment
b. finger amputation and antibiotic treatment
c. AH of the left lower extremity to identify the affected area and angioplasty with stenting, followed by
amputation of the fingers and antibiotic treatment
d. hyperbaric treatment with oxygen and antibiotics
e. amputation below the knee !

The correct answer is: amputation below the knee

On CT angiography: three areas of narrowing and 5 cm of complete occlusion on the left common iliac artery; in
addition, he has an occlusion of the 18 cm portion of the left superficial femoral artery, with contrasting the popliteal
artery and the tibial artery that is passable and intact. Recommendations?

Select one:
a. angioplasty of the iliac arteries with left stenting followed by open femoral and popliteal bypass !

b. aorto-bifemoral shunt
c. continued lifestyle changes and modification of risk factors
d. left iliac artery angioplasty and left superficial femoral artery with stenting
e. right-to-left femoral artery bypass grafting

The correct answer is: angioplasty of the iliac arteries with left stenting followed by open femoral and popliteal
bypass

The patient returns after 8 months with worsening of the pain in the calf with minimal loading and trophic ulcer at
the end of the left fourth toe. His PCI on the left is 0.37. Which of the following is now likely to be associated with
occlusive disease?

Select one:
a. infra-renal aorta

b. thoracic aorta
c. left internal iliac artery
d. left superficial femoral artery !

e. left deep femoral artery

The correct answer is: left superficial femoral artery

Finish review

◀︎ Cardiac injures Jump to... 16 ▶︎

Quiz navigation
1 2 3 4 5 6 7 8 9 10 11 12 13 14 15 16 17 18 19

Show one page at a time


Finish review

Copyright © 2022 Technomatix Ltd. | All Rights Reserved


( Distance Learning % &

" Dashboard Dashboard / My courses / 6 Surgery with pediatric surgery and oncology
/ 18 Thrombosis and embolisms of vessels, causes of development, diagnosis, differential diagnosis, treatment and
prevention. Modern possibilities of conservative treatment.
! My Training / Test 18

" Catalog

# Calendar
Started on Sunday, 2 October 2022, 9:33 AM
$ Reports State Finished
Completed on Sunday, 2 October 2022, 9:36 AM
Time taken 3 mins 16 secs
# 6 Surgery with pediatric
surgery and oncology Marks 16.00/16.00
Grade 3.00 out of 3.00 (100%)
$ Participants
A 49-year-old man with diabetes 5 days ago had laparoscopic colectomy. He is now complaining of swelling and
% Badges pain in his right calf and thigh. His life signs are normal. Which of the following is the next step for this patient?

& Competencies Select one:


a. systemic thrombolytic therapy
' Grades
b. chest CT and pelvic CT
c. determine the level of d-dimer, pulmonary angiogram, if the value of the former is elevated

d. duplex scan of lower extremity veins !

e. pulmonary angiography

The correct answer is: duplex scan of lower extremity veins

A 52-year-old woman complains of left leg pain after a cholecystectomy a few days after surgery. History of stage 3
lung cancer with colon metastases, diabetes mellitus, and secondary anemia after gastrointestinal bleeding. The
examination palpates too laterally on the left calf. The leg is warm and noticeably swollen. Which of the following is
the best treatment?

Select one:
a. anticoagulation therapy with heparin
b. installation of coffee filter !

c. anticoagulation therapy with heparin followed by warfarin

d. low molecular weight heparin


e. aspirin

The correct answer is: installation of coffee filter

A 65-year-old woman is admitted to the emergency department with a swollen right cast. She has lung cancer and
is receiving chemotherapy. GWT is suspected, the D-dimer test result is highly positive. Preliminary blood tests
show that this patient's D-dimer levels have always been above normal. What is the next step in the study of this
patient?

Select one:
a. natriuretic peptide test for the elimination of heart failure caused by chemotherapy
b. compression ultrasonography !

c. do not carry out further examination


d. venography

The correct answer is: compression ultrasonography

A 67-year-old man was replaced by a hip joint. The surgery and the immediate postoperative period went without
complications. One week after surgery, he complained of low fever and left leg pain, extending from the medial thigh
to the lithium. The examination hurts worse with dorsiflexion of the foot. There is a slight change in the color of the
left lower extremity and pain in palpation of the calf muscles. Which of the following is the best step forward?

Select one:
a. determination of D-dimer
b. compression ultrasonography !

c. perfusion-ventilation test
d. observation
e. spiral axial CT scan

The correct answer is: compression ultrasonography

Continue the thesis. Paradoxical embolism ...

Select one:
a. emboli arising in the venous system migrate throughout the body.
b. occurs in patients suffering from pulmonary hypertension and atrial septal defect. !

c. a thrombus is formed in the ascending aorta


d. a thrombus is formed in the abdominal aorta

The correct answer is: occurs in patients suffering from pulmonary hypertension and atrial septal defect.

Late effect or delay of complication of HT:

Select one:
a. lung infarction

b. paradoxical embolism
c. embolism of cerebral vessels
d. post-thrombophlebitis syndrome !

The correct answer is: post-thrombophlebitis syndrome

Most common anamnesis in patients with HRT:

Select one:
a. seizures in the lower part of the calf for several days
b. seizures in the lower part of the calf over several days that progress over time !

c. feeling tired in the calf

d. difficulty in walking longer than a few minutes due to pain in the calf
e. shooting pain below the calf

The correct answer is: seizures in the lower part of the calf over several days that progress over time

One of the most common autosomal dominant genetic mutations causing thrombophilia:

Select one:
a. antithrombin deficiency
b. Bernard Sullier syndrome
c. Leiden's factor V !

d. protein deficiency c and s

The correct answer is: Leiden's factor V

The most common cause of thrombophilia:

Select one:
a. paroxysmal nocturnal hemoglobinuria
b. true polycythemia
c. arombotic thrombocytopenic purpura.
d. antiphospholipid antibodies !

e. heparin-induced thrombocytopenia
f. thrombocytosis

The correct answer is: antiphospholipid antibodies

The most common source of paradoxical embolism:

Select one:
a. pelvic vein thrombosis
b. isolated cast vein thrombosis !

c. proximal HT
d. femoral vein thrombi

The correct answer is: isolated cast vein thrombosis

The pathogenesis of deep vein thrombosis (GVT) involves three factors known as the Virchow Triad. All of the
following factors are part of the triad, except:

Select one:
a. venous stasis
b. history of thrombosis !

c. damage to the vessel wall


d. hypercoagulopathy

The correct answer is: history of thrombosis

The patient has found GVT of the calf with spread on the right proximal vein anticoagulation therapy should be
started. Which of the following is an absolute contraindication to anticoagulant therapy?

Select one:
a. severe hypotension

b. gastrointestinal bleeding in the last 6 months


c. brain metastases
d. platelet count <20 × 103 / µl !

The correct answer is: platelet count <20 × 103 / µl

The patient is prescribed unfractionated heparin. All of the following are consequences of unfavorable heparin intake
except:

Select one:
a. sncreased partial time of thromboplastin
b. deterioration of renal failure !

c. osteoporosis
d. thrombocytopenia

The correct answer is: deterioration of renal failure

Venous thromboembolism (VTE) includes:

Select one:
a. GWT !

b. arterial thrombus
c. hypotension
d. thrombocytopenia

The correct answer is: GWT

Which of the following is the best prevention of VTE for a 33-year-old man after a diagnostic laparotomy, distal
pancreatectomy, splenectomy, and plastic perforation of the stomach, from gunshot wound to the abdomen? He had
about 3000 ml of blood loss during surgery. After surgery, he was stable in the intensive care unit with hematocrit
28%, MNV - 1.5. He currently has no signs of active blood loss.

Select one:
a. NMG !

b. 5000 units NFG


c. 5000 units NFG
d. continue prevention of DVT until the EOM is normalized
e. pneumatic compression devices

The correct answer is: NMG

Which of the patients with confirmed femoral vein thrombosis is unfractionated heparin contraindicated?

Select one:
a. a 28-year-old man with a third-degree liver rupture after a gunshot wound to the abdomen
b. 23-year-old woman in third trimester of pregnancy
c. 44-year-old woman with heparin-induced thrombocytopenia !

d. a 20-year-old man who suffered a concussion (without CT changes) as a result of a car accident 8 days ago
e. hemodynamically stable 80-year-old man with symptoms of left-sided femoral HT

The correct answer is: 44-year-old woman with heparin-induced thrombocytopenia

Finish review

◀︎ 16 Jump to... 18 ▶︎

Quiz navigation
1 2 3 4 5 6 7 8 9 10 11 12 13 14 15 16

Show one page at a time


Finish review

Copyright © 2022 Technomatix Ltd. | All Rights Reserved


( Distance Learning % &

" Dashboard Dashboard / My courses / 6 Surgery with pediatric surgery and oncology
/ 19 The vena cava superior syndrome. The vena cava inferior syndrome. Varicose veins. Postthrombophlebitis syndrome.
Causes, diagnosis, differential diagnosis, medical tactics.
! My Training / Test 19

" Catalog

# Calendar
Started on Saturday, 15 October 2022, 10:19 AM
$ Reports State Finished
Completed on Saturday, 15 October 2022, 10:22 AM
Time taken 3 mins 12 secs
# 6 Surgery with pediatric
surgery and oncology Marks 20.00/20.00
Grade 3.00 out of 3.00 (100%)
$ Participants
. Eleven years after undergoing right modified radical mastectomy, a 61-year-old woman develops raised red and
% Badges purple nodules over the right arm. What is the most likely diagnosis?

& Competencies Select one:


a. Lymphangitis
' Grades
b. Hyperkeratosis
c. Metastatic breast cancer

d. Lymphangiosarcoma !

e. Lymphedema

The correct answer is: Lymphangiosarcoma

A 25-year-old woman presents to the emergency room complaining of redness and pain in her right foot up to the
level of the midcalf. She reports that her right leg has been swollen for at least 15 years, but her left leg has been
normal. On physical examination she has a temperature of 39C0 (102.2F). The left leg is normal. The right leg is not
tender, but it is swollen from the inguinal ligament down and there is an obvious cellulitis of the right foot. What is
the patient's problem?

Select one:
a. The decompensated stage
b. Deep venous thrombosis. !

c. Thrombosis of renal veins

d. Paget-Schrotter syndrome

The correct answer is: Deep venous thrombosis.

A 45-year-old woman undergoes cardiac catheterization through a right femoral approach. Two months later, she
complains of right lower extremity swelling and notes the appearance of multiple varicosities. On examination, a bruit
is heard over the right groin. What is the most likely diagnosis ?

Select one:
a. Deep vein insufficiency.
b. Superficial venous insufficiency.
c. Femoral artery thrombosis.
d. Arteriovenous (AV) fistula. !

The correct answer is: Arteriovenous (AV) fistula.

A 45-year-old woman undergoes cardiac catheterization through a right femoral approach. Two months later, she
complains of right lower extremity swelling and notes the appearance of multiple varicosities. On examination, a bruit
is heard over the right groin. What is the most likely diagnosis?

Select one:
a. Superficial venous insufficiency

b. Femoral artery thrombosis


c. Arteriovenous (AV) fistula !

d. Pseudoaneurysm
e. Deep vein insufficiency

The correct answer is: Arteriovenous (AV) fistula

A middle-age woman has right leg and foot non pitting edema associated with dermatitis and hyperpigmentation.
The diagnosis of chronic venous insufficiency is made. What is the treatment of choice?

Select one:
a. Vein stripping

b. Skin grafting
c. Pressure-gradient stockings !

d. Valvuloplasty
e. Perforator vein ligation

The correct answer is: Pressure-gradient stockings

A middle-aged man known to have peptic ulcer disease is admitted with upper gastrointestinal (GI) bleeding. During
his hospital stay, he develops DVT of the left lower extremity. What is the most appropriate management?

Select one:
a. Inferior vena cava (IVC) filter !

b. Anticoagulation
c. Venous thrombectomy.
d. Observation
e. Thrombolytic therapy

The correct answer is: Inferior vena cava (IVC) filter

A middle-aged man known to have peptic ulcer disease is admitted with upper gastrointestinal (GI) bleeding. During
his hospital stay, he develops DVT of the left lower extremity. What is the most appropriate management?:

Select one:
a. Thrombolytic therapy
b. Inferior vena cava (IVC) filter !

c. Observation
d. Venous thrombectomy
e. Anticoagulation

The correct answer is: Inferior vena cava (IVC) filter

A newborn girl with family history of lymphedema is noted to have bilateral lower extremity swelling. What is the
diagnosis?

Select one:
a. Milroy disease !

b. Meigs’s syndrome

c. Secondary lymphedema
d. Lymphedema praecox
e. Lymphedema tarda

The correct answer is: Milroy disease

A young basketball player develops an acute onset of subclavian vein thrombosis (effort thrombosis) after heavy
exercise. What is the next step in management ?

Select one:
a. Antibiotics.
b. Active exercise of the limb.

c. Anti-inflammatory drugs.
d. Thrombolytic therapy. !

The correct answer is: Thrombolytic therapy.

A young basketball player develops an acute onset of subclavian vein thrombosis (effort thrombosis) after heavy
exercise. What is the next step in management?

Select one:
a. Thrombolytic therapy !

b. Anti-inflammatory drugs
c. First-rib resection
d. Active exercise of the limb

e. Antibiotics

The correct answer is: Thrombolytic therapy

All these venous systems are present on lower extremity EXCEPT:

Select one:
a. Lymphangitis
b. Popliteal vein !

c. Veins arising from calf perforators


d. Deep veins

e. Short saphenous vein

The correct answer is: Popliteal vein

An 18-year-old man develops a painful, swollen leg while training for the New York Marathon. There is tenderness in
the calf and ecchymosis is present. What is the most likely diagnosis?

Select one:
a. Medical lemniscus tear.
b. DVT
c. Superficial thrombophlebitis
d. Cellulitis

e. Tear of the plantaris muscle !

The correct answer is: Tear of the plantaris muscle

Four days after a hysterectomy, a 30-year-old woman develops phlegmasia cerulean dolens over the right lower
extremity. What is the most appropriate treatment ?

Select one:
a. Venous thrombectomy.
b. Local urokinase infusion.
c. Bed rest and elevation.
d. Systemic heparinization. !

The correct answer is: Systemic heparinization.

Four days after undergoing hysterectomy, a 30-year-old woman develops phlegmasia cerulea dolens over the right
lower extremity. What is the most appropri-ate treatment?

Select one:
a. Local urokinase infusion

b. Venous thrombectomy !

c. Bed rest and elevation


d. Prophylactic vena caval filter
e. Systemic heparinization

The correct answer is: Venous thrombectomy

Four days after undergoing subtotal gastrectomy for stomach cancer, a 58-year-old woman complains of right leg
and thigh pain, swelling and redness, and has tenderness on examination. The diagnosis of deep vein thrombosis is
suspected. What is the initial test to establish the diagnosis?

Select one:
a. Venous duplex ultrasound !

b. Venography
c. Impedance plethysmography
d. Radio-labelled fibrinogen
e. Assay of fibrin/fibrinogen products

The correct answer is: Venous duplex ultrasound

Four days after undergoing subtotal gastrectomy for stomach cancer, a 58-year-old woman complains of right leg
and thigh pain, swelling and redness, and has tenderness on examination. The diagnosis of deep vein thrombosis is
suspected. What is the initial test to establish the diagnosis?:

Select one:
a. Assay of fibrin/fibrinogen products

b. Impedance plethysmography
c. Radio-labelled fibrinogen
d. Venous duplex ultrasound !

e. Venography

The correct answer is: Venous duplex ultrasound

Lymphedema results from:

Select one:
a. Under the influence of intestinal helminths
b. Under the influence of infection factors

c. Mechanical insufficiency due to injury to the lymph vessels or impairment of lymph flow due to paralysis, !
blockage or inadequacy of lymph

The correct answer is: Mechanical insufficiency due to injury to the lymph vessels or impairment of lymph flow due
to paralysis, blockage or inadequacy of lymph

Tests for estimation of deep veins patency

Select one:
a. Troyanov-Trendelenburg's test.
b. Delbet-Perthes' test !

c. Hackenbruch's test.
d. Sheinis' test.
e. Talman's test

The correct answer is: Delbet-Perthes' test

The contraindications for surgical treatment of varicosity include all of the following EXCEPT

Select one:
a. Decompensation of varicosity with incompetence of valves of superficial and perforating veins !

b. Decompensative heart failure


c. Diseases of liver and kidneys with considerable disturbances of function
d. Obesity of III degree
e. Obstruction of deep veins

The correct answer is: Decompensation of varicosity with incompetence of valves of superficial and perforating
veins

The manifestations of decompensation with trophic changes of soft tissues such as indurations, hyperpigmentation,
hemosiderosis, and dermatitis of the skin are typical for:

Select one:
a. Chronic venous insufficiency of II-nd degree . !

b. The compensated stage of varicosity


c. Chronic venous insufficiency of the III-rd degree
d. Chronic venous insufficiency of the I-st degree
e. The decompensated stage of varicosity

The correct answer is: Chronic venous insufficiency of II-nd degree .

Finish review

◀︎ 18 Jump to... 19 ▶︎

Quiz navigation
1 2 3 4 5 6 7 8 9 10 11 12 13 14 15 16 17 18 19 20

Show one page at a time


Finish review

Copyright © 2022 Technomatix Ltd. | All Rights Reserved

You might also like